24.1 Flashcards

1
Q

A medication that would be acceptable to a patient who refuses all products derived
from human plasma is:

a) Prothrombinex
b) Activated factor 7
c) Fibrinogen concentrate
d) Albumin
e) anti-d

A

Factor 7 - Recombinant, made from baby hamster kidney cells

Albumin - Alburex® 5 AU (Human Albumin 50 g/L) is an Australian manufactured albumin product

Fib con - Lyophilised precipitate. manufactired from cryoprecipitate.

PCC - Prothrombinex-VF® is a lyophilised concentrate of human coagulation factors containing factors II, IX and X and a small amount of factor VII.

Red cross lifeblood.

Correct answer is rVIIa

How well did you know this?
1
Not at all
2
3
4
5
Perfectly
2
Q

An adult patient undergoing cardiac surgery exhibits excessive bleeding following
cardiopulmonary bypass. A thromboelastogram performed on their blood is shown
below. The most likely cause of the bleeding is

(ROTEM with low Extem A10 and normal Fibtem A10)

a) Platelets
b)Fibrinogen
c) FFP
d) TXA

A

Plateltes

Fibrinogen if low Fibtem
TXA if curves tail off early
FFP if MCF low

How well did you know this?
1
Not at all
2
3
4
5
Perfectly
3
Q

A term neonate is undergoing closure of gastroschisis under general anaesthesia with pressure control ventilation via an endotracheal tube. The estimated blood loss is 10 mL. Fluid therapy has been 4% albumin 40 mL/kg in addition to maintenance 10% dextrose 4 mL/kg/h. During closure of the defect, the oxygen saturation falls to 80%. The most likely cause of the desaturation is:

a) Pulmonary oedema/excessive fluids
b) Reduced Lung compliance
c) Undiagnosed congenital heart disease
4) Return to foetal circulation

A

b) Reduced Lung compliance

  • Closure of abdominal wall post gastroschisis repair leads to significantly increased abdominal compartment pressures and can splint diaphragm. May need staged closure.

Term neonate = ~3.5 kg
40ml/kg = ~140mls in
Normal blood volume 90 x 3.5 = 315ml
10ml blood loss + added environmental losses from exposed bowel
The key is the timing with closure, and to be aware that staged closures are frequently done. Most likely answer is lung complicance, and PCV which would result in a reduction in volumes on closing.

How well did you know this?
1
Not at all
2
3
4
5
Perfectly
4
Q

Phaeochromocytoma commonly presents with all of the following EXCEPT:

a) RV Hypertrophy
b) Pulmonary HTN
c) long QT
d) ST changes
e) Cardiomyopathy

A

b) Pulmonary HTN

Long QT + ST changes common
Cardiomyopathy less common but well documented
RVH possible, although more commonly LVH

How well did you know this?
1
Not at all
2
3
4
5
Perfectly
5
Q

REview Duchenne muscular dystrophy is NOT associated with:

a) Increased CK
b) Cardiomyopathy in female carriers
c) decreased Sensitivity to non-depolarising NMBs

Alternative remembered answers:

a) Reisistant to NDNMB
b) Premature death
c) Aspiration
d) Conduction abnomality in females

A

Increased sensitivity to non depolarisers

Ck -> Anaesthesia induced rhabdo
Cardio- All at-risk females, regardless of their carrier status, should be monitored for development of cardiomyopathy

How well did you know this?
1
Not at all
2
3
4
5
Perfectly
6
Q

When administered in combination with tramadol, the agent considered highest risk
for the development of serotonin syndrome is:

a) Moclobemide
b) Escitalopram
c) Desvenlafaxine
d) Tapentadol

A

Moclobemide
- Reversible MAOI

SSRIs and SNRIs are lower risk
Tapentadol - no serotonin effect

Tranylcypromine or phenylzine are irreversible blockers and would be the highest risk

How well did you know this?
1
Not at all
2
3
4
5
Perfectly
7
Q

The action of methylene blue in treating vasoplegia is mediated by:

a) Inhibits inducible NO
b) Inhibits constitutive NO
c) Inhibits guanylate cyclase
d) Agonises angiotensin II receptors
e) Something about V1 Receptors?

A

c) Inhibits guanylate cyclase

Methylene Blue acts by inhibiting guanylate cyclase, thus decreasing C-GMP and vascular smooth muscle relaxation

How well did you know this?
1
Not at all
2
3
4
5
Perfectly
8
Q

A stellate ganglion block is NOT indicated in the management of:

a) AV block
b) Resistant ventricular arrhythmia
c) PTSD
d) Scleroderma
e) Hyperhidrosis

A

AV block

CI in
- cardiac conduction block
- Glaucoma
- Anticoagulation

Indications
Complex regional pain syndrome of the head and upper limbs
Peripheral vascular disease
Upper extremity embolism
Postherpetic neuralgia
Chronic post-surgical pain
Hyperhidrosis
Raynaud disease
Scleroderma
Orofacial pain
Phantom limb
Atypical chest pain
A cluster or a vascular headache
Post-traumatic stress disorder
Meniere syndrome
Intractable angina
Refractory cardiac arrhythmias

How well did you know this?
1
Not at all
2
3
4
5
Perfectly
9
Q

Obstructive sleep apnoea in children is diagnosed with an apnoea-hypopnoea index
(AHI) of at least:

a) >1
b) >5
c) >10

A

a) >1

0 normal

Mild/mod/severe
1-5
5-10
>10

How well did you know this?
1
Not at all
2
3
4
5
Perfectly
10
Q

Neostigmine should be avoided in patients with:

a) Familial periodic paralysis
b) Myotonia congenita
c) Duchennes
d) Beckers
e) Friedrichs ataxia

A

b) Myotonia congenita

Myotonia congenita is a condition characterized by delayed relaxation of the muscles after voluntary contraction. Neostigmine can exacerbate this delayed relaxation, potentially worsening symptoms

How well did you know this?
1
Not at all
2
3
4
5
Perfectly
11
Q

A transjugular intrahepatic portosystemic shunt procedure is contraindicated in
patients with:

a) Hepatorenal syndrome
b) Refractory ascites
c) Severe TR
d) Variceal bleeding
e) Budd chiari

A

c) Severe tricuspid regurgitation (TR)

Severe TR can lead to increased right atrial pressure, which may impede the proper function of the TIPS and worsen outcomes.

Contraindications:
Severe Hepatic encephalopathy
Severe Pulmonary Htn
Severe TR
Multiple Hepatic Cysts
Coagulopathy (relative contraindication)

How well did you know this?
1
Not at all
2
3
4
5
Perfectly
12
Q

When confirming correct placement of an endotracheal tube, verifying the presence
of sustained exhaled carbon dioxide requires all the following EXCEPT:

a) CO2 rises with expiration and falls with inspiration
b) Consistent square waveform
c) Consistent or increasing amplitude of the capnogram over 7 breaths
d) Peak amplitude more than 7.5mmHg above baseline
e) Capnogram is clinically appropriate

A

Preventing unrecognised oesophageal intubation: a consensus guideline from the Project for Universal Management of Airways and international airway societies

Suggests b) is most correct answer

Verifying the presence of sustained exhaled carbon dioxide requires all the following criteria to be met (Fig. 2; [93]):
- Amplitude rises during exhalation and falls during inspiration.
- Consistent or increasing amplitude over at least seven breaths [74, 91].
- Peak amplitude more than 1 kPa (7.5 mmHg) above baseline [74, 94].
- Reading is clinically appropriate.

How well did you know this?
1
Not at all
2
3
4
5
Perfectly
13
Q

The dataset that was used to create the Eleveld TCI model did NOT include
patients who are / have:

a) Neonates
b) Elderly
c) Cirrhotic liver disease
d) End stage renal disease

A

Neonates. Eleveld designed for a wide patient pop, but not neonates.

How well did you know this?
1
Not at all
2
3
4
5
Perfectly
14
Q

The blood product that contains the highest concentration of citrate is:

a) FFP
b) RBCs
c) Platelets
d) Cryoprecipitate
e) Fibrinogen concentrate

A

a) FFP

FFP - 20mmol/l (associated with highest rate of Citrate toxicity)
- cannot find a great reference but is quoted in Citrate Toxicity During CRRT After Massive Transfusion (they then reference 1992 guidelines from Transfusion Med, 1994 article about plasma exchage, and Miller’s 2009)

Lifeblood - additive for plasmapheresis is highest concentration of 4%
- could also argue that even if derived from whole blood donation, most of the citrate likely to be in the plasma anyway and when cellular components separated from plasma it will remain (no evidence for that)

These numbers unclear source material
Platelets - 15-20mmol/L
Plasma - 13-15mmol/L
Red cells 5-7.5mm/L
Cryo 13-15mmol/L
Fib conc - nil

How well did you know this?
1
Not at all
2
3
4
5
Perfectly
15
Q

During a new pandemic, an anaesthetist refuses to provide sedation for an elective
operation due to concern that the procedure may hasten community spread of the
disease. This is the ethical principle of:

a) Beneficence
b) Non-maleficence
c) Justice
d) Conscientious objection
e) Professional autonomy

A

Primum non nocere: First, do no harm - Non maleficence

How well did you know this?
1
Not at all
2
3
4
5
Perfectly
16
Q

The anaesthetic technique associated with the highest rate of postprocedure
patency of a newly-created arteriovenous fistula is

a) Propofol TIVA
b) Brachial plexus block
c) Sedation + LA
d) Volatile

A

Regional -ie Brachial plexus

Anaesthesia Choice for Creation of Arteriovenous Fistula (ACCess) study protocol : a randomised controlled trial comparing primary unassisted patency at 1 year of primary arteriovenous fistulae created under regional compared to local anaesthesia

supraclavicular or Axillary block

How well did you know this?
1
Not at all
2
3
4
5
Perfectly
17
Q

The image below shows the arterial pressure (red, upper line) and balloon pressure
(blue, lower line) from an intra-aortic balloon pump set at 1:2 augmentation. The
point of the waveform indicated by the large green arrow is called:

a) Assisted end diastolic
b) Assisted systolic
c) Unassisted end diastolic
d) Assisted systolic

A

Assisted end diastolic

How well did you know this?
1
Not at all
2
3
4
5
Perfectly
18
Q

A patient’s true arterial oxygen saturation will be lower than a pulse oximeter
reading in the presence of:

a) Carboxy Hb
b) Sickle cell
c) Methylene blue

A

CarboxyHb
- Probe cannot differentiate between HbO and COHb

The others cause false readings

How well did you know this?
1
Not at all
2
3
4
5
Perfectly
19
Q

Organ procurement after circulatory death is generally stood down if the time from
cessation of cardiorespiratory support to circulatory death extends beyond:

a) 60min
b) 90min
c) 120min

A

90 mins

30mins
Liver
Pancreas
Heart

60mins
Kidneys

90mins
Lungs

Page 35 ANZICS statement 2.4.3 Warm ischemia time

Donate life

How well did you know this?
1
Not at all
2
3
4
5
Perfectly
20
Q

The rank of volatile anaesthetic agents from highest to lowest derived global
warming potential over 100 years (GWP100) is:

a) Nitrous, des, iso, sevo
b) Des, iso, nitrous, sevo
c) Des, nitrous, iso, sevo
d) Nitrous, des, sevo, iso

A

B

Desflurane (Des): GWP100 around 2,500-3,000
Isoflurane (Iso): GWP100 around 1,000-1,100
Nitrous oxide (Nitrous): GWP100 around 298
Sevoflurane (Sevo): GWP100 around 130-210

How well did you know this?
1
Not at all
2
3
4
5
Perfectly
21
Q

A characteristic feature of postoperative visual loss due to posterior ischaemic optic
neuropathy is:

a) Painful
b) Normal light reflexes
c) Normal fundoscopy
d) Visual inattention

A

c) Normal fundoscopy

How well did you know this?
1
Not at all
2
3
4
5
Perfectly
22
Q

The bipolar leads of a 12-lead electrocardiogram are:

a) All
b) V1-V6
c) aVL, aVR, aVF
d) I, II, III
e) None

A

D) I, II, III

3-electrode system
- Uses 3 electrodes (RA, LA and LL)
- Monitor displays the bipolar leads (I, II and III)
Life in the Fast Lane

How well did you know this?
1
Not at all
2
3
4
5
Perfectly
23
Q

The local anaesthetic with the lowest CCCNS ratio (ratio of the drug dose required
to cause cardiac collapse to the drug dose required to cause seizure) is:

a) Levobupivacaine
b) Bupivacaine
c) Lignocaine
d) Ropivacaine

A

B) Bupivacaine

CC/CNS Ratio: the ratio of the dose required to cause CVS collapse and the dose required to cause CNS toxicity (indicates the CNS is more vulnerable than CVS)

Lignocaine: 7.1
Ropivacaine: 5.0
Bupivacaine: 3.7
Levobupivacaine: **not listed

Petkov

Ropivacaine and levobupivacaine, for example, have higher CC/CNS ratios than racemic bupivacaine; therefore, it seems logical to preferentially use these drugs when long-acting LAs are desired.

Pubmed

How well did you know this?
1
Not at all
2
3
4
5
Perfectly
24
Q

The time for reversal of therapeutic dabigatran after administration of
idarucizumab 5 g is:

a) 5 mins
b) 15 mins
c) 30 mins
d) 60 mins
e) 120 mins

A

5 mins
- Essentially one circulation time

Intravenously administer the dose of 5 g (2 vials, each contains 2.5 g) as
o Two consecutive infusions or
o Bolus injection by injecting both vials consecutively one after another via syringe

Idarucizumab was administered as one 5 g intravenous infusion over five minutes

Among the 90 patients with available data, the median maximum reversal of the pharmacodynamic anticoagulant effect of dabigatran as measured by ECT or dTT in the first 4 hours after administration of 5 g idarucizumab was 100%, with most patients (>89%) achieving complete reversal. Reversal of the pharmacodynamics effects was evident immediately after administration.

FDA Product Guide

See blue book article

How well did you know this?
1
Not at all
2
3
4
5
Perfectly
25
Q

The intrinsic muscles of the larynx do NOT include:

a) Cricothyroid
b) Suprahyoid
c) Transverse arytenoid
d) Cricoarytenoid

A

b) Suprahyoid

Suprahyoid muscles are extrinsic muscles of the larynx that attach outside the laryngeal framework and assist in swallowing and other movements. Not in anatomy for anaesthetists!

The intrinsic muscles of the larynx have a threefold func-
tion: they open the cords in inspiration, they close the cords and the laryngeal inlet during deglutition, and they alter the tension of the cords during speech.
They comprise the posterior and lateral cricoarytenoids,
the interarytenoids and the aryepiglottic, the thyroarytenoid, the thyroepiglottic, the vocalis and the cricothyroid muscles.

Anatomy for Anaesthetists

How well did you know this?
1
Not at all
2
3
4
5
Perfectly
26
Q

When interpreting an arterial blood gas, a high serum anion gap is consistent with:

a) lithium toxicity
b) Salicylate toxiticy
c) Hypercholeraemia
d) Hypoalbuminaemia
e) Hypercalcaemia

A

b) Salicylate toxicity

Salicylate toxicity can cause an elevated serum anion gap due to the production of organic acids (salicylic acid and its metabolites) that are not measured by the standard anion gap calculation. This leads to an increased anion gap metabolic acidosis.

HAGMA results from accumulation of organic acids or impaired H+ excretion

Causes (LTKR)
Lactate
Toxins
Ketones
Renal

Causes (CATMUDPILES)
CO, CN
Alcoholic ketoacidosis and starvation ketoacidosis
Toluene
Metformin, Methanol
Uremia
DKA
Pyroglutamic acidosis, paracetamol, phenformin, propylene glycol, paraladehyde
Iron, Isoniazid
Lactic acidosis
Ethylene glycol
Salicylates

NAGMA results from loss of HCO3- from ECF

Causes (CAGE)
Chloride excess
Acetazolamide/Addisons
GI causes – diarrhea/vomiting, fistulae (pancreatic, ureters, billary, small bowel, ileostomy)
Extra – RTA

Causes (ABCD)
Addisons (adrenal insufficiency)
Bicarbonate loss (GI or Renal)
Chloride excess
Diuretics (Acetazolamide)

LITFL

How well did you know this?
1
Not at all
2
3
4
5
Perfectly
27
Q

The Glasgow Blatchford score is used to risk stratify:

a) Pulmonary haemorrhage
b) Traumatic intraperitoneal haemorrhage
c) PPH
d) SAH
e) UGI bleed

A

e) UGI bleed

Stratifies upper GI bleeding patients who are “low-risk” and candidates for outpatient management. Use for adult patients being considered for hospital admission due to upper GI bleeding.

Components: haemoglobin, BUN, initial systolic BP, heart rate > 100, melena present, recent syncope, hepatic disease history, cardiac failure present.

Med-Calc

How well did you know this?
1
Not at all
2
3
4
5
Perfectly
28
Q

In a male patient with quadriplegia undergoing a rigid cystoscopy, the optimal
choice of anaesthesia to prevent autonomic dysreflexia is

a) Spinal
b) Epidural
c) GA with volatile at 1 MAC
d) Topical only

A

a) Spinal

Elective surgery.Urological. Recurrent urinary tract infections and long-term catheterization increase the risk of bladder cancer. Cystoscopy is a common procedure as is insertion of suprapubic catheters and botox injections for the management of neuro- pathic bladders.

Spinal anaesthesia is safe in patients with CSCI and is an effect- ive way of abolishing ADR15 and spasms. Spinal anaesthesia is becoming a widely accepted technique in patients with pre-exist- ing spinal cord pathology and is routinely used in Stoke Mande- ville Hospital, with a low dose (1.5–2 ml) hyperbaric bupivacaine 0.5%, for most procedures. Spinals can be challenging to site because of poor positioning as a result of spasms and contractures, the presence of spinal metal work, and bony deformities.

The effectiveness and the level of the block are difficult to ascertain. The loss of the Babinski reflex and a change in tone from spasticity to flaccid paralysis indicate an established block; although the height of the block remains difficult to assess. The anaesthetist must be vigilant for the signs and symptoms of a total spinal block.

Epidural anaesthesia has been demonstrated to be effective in reducing ADR in labouring women; however, it is less reliable for general and urological surgical procedures.

I asked a boss about this - he said if previous autonomic dysreflexia definitely needs an anaesthetic!

Perioperative management for patients with a chronic spinal cord injury. BJA 2015

How well did you know this?
1
Not at all
2
3
4
5
Perfectly
29
Q

Interference with pacemaker function can result from all of the following EXCEPT:

a) RF ablation
b) High volume ventilation
c) Peripheral nerve stimulator
d) CT
e) Diathermy

A

d) CT

British Heart Rhythm Societies guidelines

How well did you know this?
1
Not at all
2
3
4
5
Perfectly
30
Q

A neonate with a postmenstrual age of 34 weeks (born at 26 weeks) and weighing
2 kg is undergoing retinal laser therapy under general anaesthesia. The oxygen
saturation is 92% on the following ventilator settings: FiO2 0.4; inspiratory pressure
15 cmH2O; PEEP 5 cmH2O; rate 24 breaths per minute. The most appropriate
course of action is to:

a) Increase FiO2 to 100
b) Suction tube
c) Increase PEEP to 7
d) Recruitment breath at 30cmh2o
e) Do nothing

A

E: Do nothing

Targets for premature babies:
* Volume-targeted or pressure-limited mode targeting tidal volumes of 5 ml kg1
* Ventilatory frequency: 30-60 bpm
* PEEP: 6-8 cmH2O
* Titrate above to maintain normocapnia or mild hypercapnia
* Titrate FIO2 to achieve SpO2 90-95%.

RCH guidelines

How well did you know this?
1
Not at all
2
3
4
5
Perfectly
31
Q

When auscultating the heart the Valsalva manoeuvre will increase the murmur
intensity of:

a) AS
b) MS
c) MR
d) MVP
e) VSD

A

Mitral valve prolapse

Valsalva increases the strength of murmurs due to hypertrophic obstructive cardiomyopathy and mitral valve prolapse. It decreases the intensity of aortic stenosis, mitral stenosis, aortic regurgitation, mitral regurgitation, and ventricular septal defects.
OPPOSITE IS TRUE FOR SQUATTING (Increases preload)

How well did you know this?
1
Not at all
2
3
4
5
Perfectly
32
Q

The most appropriate order of blood products transfused sequentially through the
same blood administration set is:

A) RBC - plasma - plts
B) RBC - plts - plasma
C) Plasma - RBC - plts
D) Plts- RBC -plasma

A

D) Plts- RBC -plasma

according to Lifeblood guidelines, platelets MUST be given before RBC if in the same line, as red cell debris will trap platelets; platelets and plasma can be sequential through the same set; as platelets take a long time to transfuse, it makes sense to first transfuse plasma (fast), then platelets, then red cells

How well did you know this?
1
Not at all
2
3
4
5
Perfectly
33
Q

The breathing system shown in the accompanying picture is an example of
Mapleson:

a) A
b) B
c) C
d) D
e) F

A

c

How well did you know this?
1
Not at all
2
3
4
5
Perfectly
34
Q

In an anaesthetised patient with anaphylaxis, cardiac compression should be
initiated at a systolic blood pressure of less than:

a) 40
b) 60
c) 80
d) 100
e) 120

A

a) 40

40; if 50 was there the answer would be 50

NAP 6 says CPR if SBP<50mmHg
ANZAAG says 50mmHg

How well did you know this?
1
Not at all
2
3
4
5
Perfectly
35
Q

The muscle recommended for neuromuscular monitoring by the 2023 American
Society of Anesthesiologists practice guidelines is the:

a) Adductor Pollicis
b) Flexor pollicis longus
c) Flexor hallucis brevis
d) Corrugator supercilii
e) Orbicularis oculi

A

A - Adductor Pollicis - Usual site for NMT

Correct on ASA website

How well did you know this?
1
Not at all
2
3
4
5
Perfectly
36
Q

A single intraoperative dose of 8 mg dexamethasone compared to 4 mg results in:

a) No difference in analgesia
b) No difference in PONV
c) No difference in BSL
d) Increased surgical site infection

A

B is the answer

Check 4th consensus guidelines

Does show better analgesia

PADDI Trial (Monash and ANZCA) 2021
No difference in infection with dex 8 vs placebo

Anaesthesiology Nov 2021, Vol 135, issue 5 - article by Aus anaesthesiologists

A higher dose
- Will cause more hyperglycaemia in DM patients but not clinically/statiscally significant
- Will improve PONV for 72 hours = possibly
- Some studies show this can improve analgesia - ortho, ent cases particularly
8mg dose recommended
Was the question related to addition of dex in block - Korean study compared 4vs8 in 2018

How well did you know this?
1
Not at all
2
3
4
5
Perfectly
37
Q

You are undertaking an ultrasound guided pericapsular nerve group (PENG) block
for hip surgery. In the accompanying image, the structure labelled with the arrow is
the:

a) Psoas Tendon (This)
b) Iliacus
c) Sartorius

A

Add picture of peng block (can’t from my account)

a) Psoas Tendon (This)

How well did you know this?
1
Not at all
2
3
4
5
Perfectly
38
Q

The tooth most commonly damaged during direct laryngoscopy is the:

a) Right maxillary central incisor
b) Left maxillary lateral incisor
c) Left maxillary Central incisor
d) Right maxillary lateral incisor

A

c) Left maxillary Central incisor

BJA education article - 2016 and Aagbi

Left max central incisor most common from blade (fulcrum) 32%
Right max central - 19%
Right third molar if posterior injury

Periodontitis and cvs risk link
Reminder to link poor dentition with other risk factors - meth/smoking/poor diet/autoimmune conditions and their anaesthetic impact
1:4500 risk (Rcoa)

How well did you know this?
1
Not at all
2
3
4
5
Perfectly
39
Q

In the event of an electrical fire in the operating room, the correct fire extinguisher
type to use is:

a) Dry powder
b) Wet
c) Chemical
d) CO2

A

CO2

Pull/Aim/Squeeze/Sweep
Don’t use fire blankets - concentrated heat on patient
Saline or water for body cavity fire
Dry powder and chemical can leave residues that could damage equipment

How well did you know this?
1
Not at all
2
3
4
5
Perfectly
40
Q

According to the ISO colour code for medical gas cylinders, Entonox is indicated by

a) Blue/ White
b) Yellow
c) Black
d) Grey

A

a) Blue/ White

Blue and white shoulder
White bottle
Pre 2004 made cylinder is blue

How well did you know this?
1
Not at all
2
3
4
5
Perfectly
41
Q

During resuscitation of a newborn, the heart rate is noted to be 50 beats per minute
despite optimal ventilation and chest compressions. The next step in management
is to give intravenous adrenaline:

a) 0.1-0.3ml/kg 1:1000
b) 0.5-1ml/kg 1:10000
c) 0.1-0.3ml/kg 1:10000
d) 0.1-0.3ml/kg 1:100000

A

C

Neoresus
10-30mcg/kg

How well did you know this?
1
Not at all
2
3
4
5
Perfectly
42
Q

An adult weighing 80 kg has sustained full-thickness burns to 40% of their body.
The recommended volume of fluid resuscitation in the first 24:

a) 9600ml
b) 16000ml
c) 6400ml

Actual exam options gave 3ml/kg/bsa to 4ml/kg/bsa range

A

3 * 40 * 80 = 9600
4 * 40 * 80 = 12800

Parkland seems to be trending toward 3ml these days rather than 4

Deranged physiology key points
Urine output as end goal - risk of fluid creep with same
Albumin reduces total volume of resus but not difference to survival
Hypertonic fluids - increased mortality and AKI

Other formula
Brooke
Evan’s
Monafo
Shriner’s -paeds
Galvestons - paeds

How well did you know this?
1
Not at all
2
3
4
5
Perfectly
43
Q

In a can’t intubate, can’t oxygenate (CICO) scenario when using a 14G cannula
and a Rapid-O2 oxygen delivery device, the initial rescue breath should be:

a) 2 seconds, 10L O2
b) 4 seconds, 10L O2
c) 2 secs 15L
d) 4 secs 15L

A

d) 4 secs 15L

Initial breath 4 seconds @ 15L (rate is 250ml/s i.e. total delivered in 4 seconds = 1L)

If no improvement in SpO2 after 30 seconds give another 2 second breath

Subsequent breaths once sats fall by 5% from maximum Spo2 achieved with initial jet ventilation breath = 2 secs (I.e. 500ml)

How well did you know this?
1
Not at all
2
3
4
5
Perfectly
44
Q

The maximum recommended cumulative dose of Intralipid 20% for the treatment of
local anaesthesia systemic toxicity is:

a) 8ml/kg
b) 9ml/kg
c) 12ml/kg

A

c) 12ml/kg

Intralipid 20% treatment
Initial bolus 1.5ml/kg (repeat up to Max 3 times 5 mins apart

Infusion 15ml/kg /hr

Max cumulative dose = 12 ml/kg

How well did you know this?
1
Not at all
2
3
4
5
Perfectly
45
Q

The minimum age in years for in vitro contracture testing for suspected malignant
hyperthermia is

a) 6
b) 8
c) 10
d) 12

A

10

All current Australian and New Zealand laboratories follow the guidelines of the European Malignant Hyperthermia Group for In Vitro Contracture Testing.

The EMHG guidelines are summarised as follows:

Age and Weight

The minimum weight limit for Australian and New Zealand laboratories is 30 kg and the minimum age for IVCT is 10 years.
(Emhg actually says min age for muscle biopsy is 4 yrs but lab’s should not test children under 10 yrs without relevant control data)

IVCT details

The biopsy should be performed on the quadriceps muscle (eithervastus medialisorvastuslateralis), using local (avoiding local anaesthetic infiltration of muscle tissue), regional, or trigger-free general anaesthetic techniques.

The muscle samples can be dissected in vivo or removed as a block for dissection in the laboratory within 15 minutes.

The time from biopsy to completion of the tests should not exceed 5 hours.

Muscle specimens should measure 20-25 mm in length and at least four tests should be performed each one using a fresh specimen.

The tests should include a static cumulative caffeine test and a dynamic or static halothane test.

The results should be reported as the threshold concentration, which is the lowest concentration of caffeine or halothane that produces a sustained increase of at least 2 mN (0.2 grams) in baseline force from the lowest force reached.

How well did you know this?
1
Not at all
2
3
4
5
Perfectly
46
Q

A medication that should be avoided in a patient with thyroid storm is:

a) Aspirin
b) Propranolol
c) Potassium Iodide
d) PTU

A

NSAIDS/aspirin should be avoided as it displaces thyroxine from protein and subsequently increases free T3 and T4 levels.

Thyroid storm

General measures
Cooling
IVF +/- glucose
Paracetamol
Propranolol

Specific
Hydrocortisone 200 mg QID IV
PTU
after PTU sodium iodide/lugols iodine

How well did you know this?
1
Not at all
2
3
4
5
Perfectly
47
Q

A patient with a perioperative troponin rise above normal, chest pain, left ventricular
anterior regional wall motion abnormality, and atheroma without thrombus
occluding 70% of the left anterior descending coronary artery has had a/an

NSTEMI
STEMI
Unstable angina
Acute myocardial injury
Chronic myocardial injury
Type 1 MI
Type 2 MI

A

NSTEMI

MINS: MI/ischemic myocardial injury that doesn’t fulfill MI defn

MI: Myocardial injury with rise/fall cTn above 99th percentile of upper ref limit within 30 days post op plus at least one of:
Ischemic symptoms
New ischemic ECG changes
New path Q waves on ECG
Imaging evidence of myocardial ischemia
Angiographic/autopsy evidence of coronary thrombus

How well did you know this?
1
Not at all
2
3
4
5
Perfectly
48
Q

Regarding sex differences in the incidence of connected consciousness (ability to
respond to command during general anaesthesia) in adults after tracheal intubation
as measured by the isolated forearm technique:

a) Higher in females due to lower propofol ml/kg dose
b) Higher in females despite same dose propofol
c) Higher in males due to lower propofol ml/kg dose
d) Higher in males despite same propofol dose
e) No sex difference

A

B) higher in females despite same dose propofol

BJA Feb 2023

https://www.bjanaesthesia.org/article/S0007-0912(22)00192-1/fulltext

Females (13%, 31/232) responded more often than males (6%, 6/106). In logistic regression, the risk of responsiveness was increased with female sex (odds ratio [ORadjusted]=2.7; 95% confidence interval [CI], 1.1–7.6; P=0.022) and was decreased with continuous anaesthesia before laryngoscopy

*supplementary table shows dosing between female and male responders vs non responders and dosing is the same

How well did you know this?
1
Not at all
2
3
4
5
Perfectly
49
Q

A patient who underwent a thoracotomy six months ago reports shooting pain on
the chest wall occurring without any trigger. This is known as:

Post thoracotomy pain syndrome

A

IASP Post-thoracotomy pain syndrome:
“Pain that recurs or persists along a thoracotomy incision at least two months following the surgical procedure” in general it is burning or stabbing pain with dysesthesia thus shares many features of neuropathic pain.

Dysesthesia: unpleasant abnormal sensation spontaneous or evoked

How well did you know this?
1
Not at all
2
3
4
5
Perfectly
50
Q

In Australia and New Zealand, a return to practice program is recommended after an absence from consultant anaesthetic practice for more than:

a) 3 months
b) 6 months
c) 9 months
d) 12 months

A

12 months

How well did you know this?
1
Not at all
2
3
4
5
Perfectly
51
Q

In this ultrasound image, the cricothyroid membrane is at the position marked

A
B
C
D
E

A

C

How well did you know this?
1
Not at all
2
3
4
5
Perfectly
52
Q

A superficial cervical plexus block will block all of the following nerves EXCEPT the:

a) Lesser occipital
b) Greater occipital
c) Greater auricular
d) Transverse cervical
e) Supraclavicular

A

Greater occipital

How well did you know this?
1
Not at all
2
3
4
5
Perfectly
53
Q

A drug which is unlikely to interfere with skin testing is oral:

a) Diphenhydramine
b) Amitriptyline
c) Prednisolone
d) Risperidone
e) Ranitidine

A

Risperidone

Avoid antihistamines and steroids
TCAs known to interfere

Mayo clinic website

See allergy.org.au - risp mentioned in appendix b as a med that may need held

How well did you know this?
1
Not at all
2
3
4
5
Perfectly
54
Q

According to the ANZCA guideline on fatigue risk management in anaesthesia practice the duration of an ideal nap is:

a) 10-20
b) 20-30
c) 30-40
d) 40-50

A

20-30 mins

ANZCA PG43a

How well did you know this?
1
Not at all
2
3
4
5
Perfectly
55
Q

A 39-year-old requires anaesthesia for a laparoscopic cholecystectomy. They have a history of mastocytosis and have never had an anaesthetic in the past. The non-depolarising muscle relaxant to avoid using is:

a) Atracurium
b) Cisatracurium
c) Pancuronium
d) Rocuronium
e) Vecuronium

A

Atrac - histamine release is bad.

more Mastocytosis Info would be good

How well did you know this?
1
Not at all
2
3
4
5
Perfectly
56
Q

A healthy woman with an uncomplicated pregnancy has an American Society of Anesthesiologists (ASA) Physical Status classification of:

a) 1
b) 2
c) 3
d) 4

A

2

How well did you know this?
1
Not at all
2
3
4
5
Perfectly
57
Q

NP: The antibiotic considered safest to be administered to a patient with myasthenia gravis in the perioperative period is:

a) Vancomycin
b) Gentamycin
c) Erythromycin
d) Flucloxacillin
e) Ciprofloxacin

A

d) Flucloxacillin

Black box warning for fluoroquinolones (ciprofloxacin)

Probably also avoid
Aminoglycosides (Amikacins/gentamicin/streptomycin) and tobramycin although TOBRAMYCIN probably least problematic of these.
Macrolides (erythromycin)

These antibiotics have not been shown to cause many problems for MG patients
Tetracycline (doxycycline, minocycline) – this may worsen MG
Sulfonamides (Bactrim), Penicillin – causes rare cases, usually not a problem for majority of MG patients

https://myastheniagravis.org/mg-and-drug-interactions/#:~:text=These%20antibiotics%20have%20black%20box,Ketek%20(telithromycin)

https://myasthenia.org/Portals/0/Cautionary%20Drugs.pdf

How well did you know this?
1
Not at all
2
3
4
5
Perfectly
58
Q

The clinical laser type with the greatest tissue penetration is:

a) Argon
b) Nd:yag
c) Er:yag
d) Co2
e) Holmium

A

b) Nd:yag

Modified Question: this question asks Greatest, old asks least

Least = Er:yag
Most = Nd:Yag

Er:yag (Erbrium-Yag) used in dermatology which is the least penetrative

CO2 laser has very little penetration (~ 10micrometres), as it has a wavelength of 10 600nm.
Helium-Neon laser also has very little penetration.

Nd:YAG is the most powerful, with a penetration of 2-6mm, as it has a wavelength of 1064nm.

Argon penetration of 0.5mm

How well did you know this?
1
Not at all
2
3
4
5
Perfectly
59
Q

The accompanying image is obtained while doing an ultrasound guided erector spinae plane block at the level of the transverse process of the fourth thoracic vertebra. The muscle marked by the arrow is the:

a) Traps
b) Rhomboids
c) Erector spinae
d) Latissimus Dorsi

A

c) Erector spinae

How well did you know this?
1
Not at all
2
3
4
5
Perfectly
60
Q

Risk factors for delirium after hip fracture surgery include all EXCEPT

a) Frailty
b) Age
c) GA vs Neuraxial technique
d) Male Sex

A

c) GA vs Neuraxial
Neuraxial versus general anesthesia in elderly patients undergoing hip fracture surgery and the incidence of postoperative delirium: a systematic review and stratified meta-analysis:
This meta-analysis did not find any statistically significant difference in POD incidence between NA and GA groups or in any subgroup analyses. There was no difference in delirium incidence regardless of inclusion or exclusion of patients with pre-existing dementia or preoperative delirium

a) Frailty, b) Age -> risk factors
Most notably, neck of femur fracture repair is associated with up to 70% risk of postoperative delirium. There are several explanations: a neck of femur fracture is commonly associated with frail older patients; perioperative pain is a significant issue; and the surgery is usually done in an emergency setting with limited opportunity for preoperative optimisation
BJA

d) Male sex -> risk factor
Male sex associated with increased risk of delirium, multiple studies on Google

How well did you know this?
1
Not at all
2
3
4
5
Perfectly
61
Q

The effects of empagliflozin include a decrease in:

a) Ketone production
b) Intravascular volume
c) Serum creatinine
d) Glycosuria

A

b) Intravascular Volume

Common Adverse Effects
- genital infections (eg vulvovaginal candidiasis, balanitis)
- polyuria
- dysuria
- UTI
- dyslipidaemia
- hypoglycaemia (when used with a sulfonylurea or insulin)
- increased haematocrit
- constipation
- nausea
- thirst
- renal impairment, eg increased serum creatinine (related to volume depletion, generally occurs early in treatment and is reversible)

Australian Medicines Handbook

62
Q

Oral naltrexone should be ceased preoperatively for:

a) 24 hours
b) 48 hours
c) 72 hours
d) 96 hours

A

NAOMI 72 hours
ANZCA Blue Book 2023

Oral naltrexone should be stopped at least 24 hours and ideally 72 hours prior to elective surgery.
And there is a lack of instruction re Contrave- so best to stop 72 hours prior.
And limited evidence re low dose naltrexone for chronic pain - so for consistency blue book says 72 hours.

Caution increased opioid sensitivity in patients using perioperative naltrexone.

63
Q

A medication that has NOT been associated with arrhythmogenic potential in patients with Brugada syndrome is:

a) Propofol
b) Thiopentone
c) Amiodarone
d) Ketamine

A

B Thiopentone

BJA article 2018

Propofol infusions have been associated with a brugada like ECG.

https://www.brugadadrugs.org/avoid/

64
Q

Borders of the anterior triangle of the neck DO NOT include the:

a) Inferior angle of mandible
b) Middle third of clavicle
c) Sternocleidomastoid muscle
d) Midline neck

A

b) Middle third of clavicle

Anterior triangle contains IJ
Superiorly: inferior border of the mandible.
Laterally: anterior border of the sternocleidomastoid.
Medially: sagittal line down the midline of the neck.

Posterior triangle contains EJ
Anterior: posterior border SCM
Posterior: anterior border trapezius
Inferior: middle third clavicle

StatPearls
Anatomy, Head and Neck, Neck Triangle

65
Q

In a patient presenting with an Addisonian crisis, the electrolyte disturbances MOST LIKELY to be seen are:

a) Hypocalcaemia, hyperkalaemia, hyponatraemia
b) High BSL, hyperkalaemia, hyponatraemia
c) Low BSL, hyperkalaemia, hyponatraemia
d) Hypercalcaemia, hyperkalaemia, hyponatraemia

A

c) Low BSL, hyperkaelamia, hyponatraemia

Adrenal crisis is a medical emergency and should be considered in any patient presenting with one or more of the following symptoms:
* altered consciousness
* circulatory collapse
* hypoglycaemia
* hyponatraemia
* hyperkalaemia
* seizures
* history of steroid use/withdrawal
* any clinical features of Addison disease

Adrenal crisis may be precipitated by stress, sepsis, dehydration or trauma; clinical features may be modified accordingly. In patients with known adrenal insufficiency, nonadherence with therapy, inappropriate cortisol dose reduction or lack of stress related cortisol dose adjustment can cause adrenal crisis.

Aus Family Physician - RACGP

66
Q

A local anaesthetic agent that is considered safe to use in a patient with glucose-6-phosphate dehydrogenase deficiency is:

a) Articaine
b) Bupivacaine
c) Lignocaine
d) Prilocaine
e) Benzocaine

A

Bupivicaine

Avoid
- Lignocaine/lidocaine
- Prilocaine

Considering the specificity of patients with G6PD deficiency, attention should be given to the choice of local anesthetic, because for instance lidocaine and prilocaine are not recommended, while bupivacaine is shown to be safe to use.

Management of Anesthesia and Perioperative Procedures in a Child with Glucose-6-Phosphate Dehydrogenase Deficiency
J Clin Med. 2022 Nov

Also avoid methylene blue (prev Q)

Could only find
- don’t give lignocaine
- can give bupivacaine

Also found don’t give articaine, prilocaine or benzocaine
https://cdho.org/factsheets/glucose-6-phosphate-dehydrogenase/#:~:text=Local%20anaesthetic%20agents%20(e.g.%2C%20prilocaine,9%20in%20G6PD%20deficient%20persons.

67
Q

Steph The ANZAAG-ANZCA guideline for management of resistant hypotension during perioperative refractory anaphylaxis in an adult includes all of the following
EXCEPT:

a) Fluid bolus 20ml/kg
b) Continue adrenaline
c) Noradrenaline infusion
d) Vasopressin bolus
e) Glucagon

A

a) its 50ml/kg bolus

Resistant Hypotension
* Additional IV fluid bolus 50 mL/kg
* Continue Adrenaline Infusion
* Add second vasopressor
* Consider CVC
* TOE/TTE
* Cardiac bypass/ECMO if available

Adult Recommendations
- Additional IV fluid bolus 50 mL/kg
- Noradrenaline Infusion 3 – 40 mcg/min (0.05 - 0.5 microg/kg/min) and/or
- Vasopressin bolus 1– 2 units then 2 units per hour
- If neither available use either
Metaraminol or Phenylephrine
- Infusion Glucagon 1– 2 mg IV every 5 min until response: draw up and administer IV (Counteract β blockers)

Anaphylaxis during Anaesthesia
Refractory Management
ANZAAG-ANZCA guideline

Also useful; resistant bronchospasm
* Consider:
- Oesophageal intubation
- Circuit malfunction
- Airway device malfunction
- Tension pneumothorax
* Continue Adrenaline Infusion
* Add alternative bronchodilators

Adult Recommendations
Salbutamol
* Metered Dose Inhaler 12 puffs (1200 microg)
* IV bolus 100-200microg
+/- infusion 5-25microg/min Magnesium 2 g (8 mmol) over 20 minutes
Consider Inhalational Anaesthetics and Ketamine

68
Q

The abnormalities seen in the electrocardiogram below are consistent with:

a) Hypercalcaemia
b) Hypermagnasaemia
c) Hyperphosphataemia
d) Hypokalaemia
e) Hyperkalaemia

A

NAOMI

ECG features of hypokalemia:

Increased P wave amplitude
Prolongation of PR interval
Widespread ST depression and T wave flattening/inversion
Prominent U waves (best seen in the precordial leads V2-V3)
Apparent long QT interval due to fusion of T and U waves (= long QU interval)

69
Q

The following supraglottic airway devices allow direct intubation EXCEPT for the:

a) Classic
b) iGel
c) Auragain
d) Supreme
e) Proseal

A

d) Supreme

Classic has the internal bands that need cutting ?

iGel clearly used for direct intubation.

I can’t see anything about “direct intubation” with the supreme but can find via bougie and aintree exchange.

Auragain allows: The Ambu® AuraGain™ (Ambu A/S, Ballerup, Denmark) is a new anatomically curved, single-use supraglottic airway device launched in June 2014, that features an integrated gastric access port and direct intubation capability using a standard endotracheal tube ideally guided by a flexible videoscope.

Proseal can’t find anything

Supraglottic airway devices as conduits for tracheal intubation
There are a number of SGAs that allow direct passage of an adult-sized ETT, including the LMA Fastrach™, the LMA Classic Excel™, the LMA cTrach™ (LMA™ North America, Inc., San Diego, CA, USA), the Air-Q™ (Mercury Medical®, Clearwater, FL, USA), the Ambu® Aura-i™, the Ambu® AuraOnce™ (Ambu Inc. Glen Burnie, MD, USA), the i-gel™ (Intersurgical Ltd., Liverpool, NY, USA), and the Elisha airway device (Elisha Medical Technologies, Ltd., Katzrin, Israel).4-9 An adult ETT can be placed directly into the trachea via these SGAs, either blindly or with bronchoscopic guidance. However, these SGAs designed for intubation may not be readily available in the operating room,10 and many anesthesiologists are not familiar with using them as stand-alone airways or intubating devices.

https://link.springer.com/article/10.1007/s12630-012-9714-8 (its old from 2012 but lists classic as a no.

There are other SGAs in which passage of an adult full-sized ETT into the trachea may be problematic, including the LMA Classic™, the LMA-Unique™, the double lumen SGAs (i.e., the LMA ProSeal™ and LMA Supreme™), and other SGAs. While they do not permit direct ETT intubation, these SGAs are readily available in the operating room and are commonly used as a stand-alone airway device. The small internal diameter (ID) of the airway lumen in these SGAs prevents an adult full-sized ETT (7.0 mm or larger) from passing through. In addition, the length of the ETT may be too short to reach mid-trachea (Fig. 1). This increases the risk of vocal cord compression and dislodging when the SGA is removed.1-3,11,12 In this case, the use of introducers or catheters is necessary to achieve tracheal intubation. In this article, we review the use of the following devices: 1) an Aintree Intubation Catheter, 2) a guidewire catheter, 3) a gum elastic bougie, and 4) a small ETT.

Many extraglottic airway devices allow the direct passage of an adult-sized tracheal tube, but this is not possible with the LMA-Supreme(TM) . We evaluated the feasibility of using the LMA-Supreme(TM) as a conduit for intubation in patients with known difficult airways.

CJA 2012 - supreme requires use of guidewire/AIC/fibrescope

70
Q

The MELD-Na (Model for End-Stage Liver Disease-Sodium) score includes all of the following parameters EXCEPT:

a) Bilirubin
b) INR
c) Albumin
d) Creatinine

A

Albumin

MELD uses the following parameters:
- Bilirubin
- INR
- Creatinine
- [Hyponatraemia]
○ Part of the MELD-Na score update in 2016
○ Sodium (Na) Values < 125 are set to 125 and values >137 are set to 137
4 MELD levels are:
- >/=25 (gravely ill)
- 24-19
- 18-11
- </=10

In patients who have undergone abdominal surgery an elevated MELD score was a better predictor of poor perioperative outcome than Child-Pugh Classification
- MELD score >15 should avoid elective surgery

Calculation:
MELD =
3.8loge(serum bilirubin [mg/dL]) + 11.2loge(INR) + 9.6*loge(serum creatinine [mg/dL]) + 6.4

MELD-Na =
MELD + 1.32 * (137-Na) - [0.033*MELD * (137-Na)]

71
Q

Postdural puncture headache in obstetric anaesthesia is associated with a greater likelihood of all of the following EXCEPT:

a) Sheehan’s
b) Cortical vein thrombosis
c) Bacterial meningitis
d) Postpartum depression

A

repeat: Sheehans

https://www.uptodate.com/contents/post-dural-puncture-headache

Complications of PDPH
1. Chronic Back pain
2. Hearing loss
3. Acute onset headache consider pneumopcephalus headache
4. Persistent headache
5. Increased risk of subdural haematoma
6. postpartum depression
7. bacterial meningitis
8. Reversible cerebral vasoconstriction syndrome (RCVS)
9. Posterior reversible encephalopathy syndrome (PRES)

72
Q

Cyclooxygenase-2 (COX-2) inhibitors in pregnancy are considered:

a) Not safe
b) Safe
c) Safe only in 3rd trimester
d) Safe after 1st trimester up to 48hrs prior to delivery

A

REPEAT with slightly reworded answers
A is safest answer.

Previous iterations of this Q have been controversial. NSAIDs technically safe in first trimester, but not in third (post 32 weeks) as can cause premature duct closure.

a. Not safe
or
safe only in 1st trimester

While relatively safe in early and mid pregnancy, NSAIDs can precipitate fetal cardiac and renal complications in late pregnancy, as well as interfere with fetal brain development and the production of amniotic fluid; they should be discontinued in gestational wk 32

APMSE

73
Q

A healthy woman is admitted to the obstetric unit with threatened preterm labour at 29 weeks gestation. Her blood pressure is 140/80 mmHg. A magnesium sulfate infusion is indicated for the purpose of:

a) Foetal neuroprotection
b) Treat BP
c) eclampsia prevention
d) Tocolysis

A

A - Foetal neuroprotection

https://www.kemh.health.wa.gov.au/~/media/HSPs/NMHS/Hospitals/WNHS/Documents/Clinical-guidelines/Obs-Gyn-Guidelines/Preterm-Labour-Magnesium-Sulphate-for-Neuroprotection-of-the-Fetus.pdf?thn=0#:~:text=MgSO4%20is%20only%20given%20to,4%20hours%20prior%20to%20delivery.

74
Q

The commonest symptom or sign of uterine rupture during attempted vaginal birth after caesarean is:

a) Pain between contractions
b) CTG persistent foetal bradycardia
c) Variable decels on CTG
d) PV Bleeding

A

Fetal bradycardia

No idea - commonest sign is pv bleeding and fetal brady (non specific) and both answers are there!

Mentioned on MELB course that foetal Brady

75
Q

A 50-year-old has had a headache for the last month which is relieved by lying flat. They have had no medical procedure to their spine such as epidural, spinal or lumbar puncture. Their brain magnetic resonance (MR) imaging scan shows diffuse meningeal enhancement and brain sagging. The neurologist suspects spontaneous intracranial hypotension and asks you to do an epidural blood patch. No spinal imaging has been performed to confirm a cerebrospinal fluid (CSF) leak. You should

A do LP to measure pressure if low do lumbar patch
B do blood patch at lumbar level with no further investigation
C do spine imaging if CSF leak present do blood patch at level
D do spine imaging if CSF leak present do lumbar blood patch
E refuse to do blood patch

A

D do spine imaging if CSF leak present do lumbar blood patch

76
Q

Following scoliosis surgery, a patient exhibits neurological changes in both legs. There is loss of power and reduced pain and temperature sensation. Proprioception and vibration sense are intact. The most likely mechanism of injury is:

a) Misplaced pedicle screw
b) Anterior spinal artery syndrome
c) Posterior spinal artery syndrome
d) Brown-Sequard syndrome

A

STEPH a) Anterior spinal artery syndrome

Anterior spinal artery syndrome usually includes tracts in the anterior two-thirds of the spinal cord, which include the CSTs, the spinothalamic tracts, and descending autonomic tracts to the sacral centers for bladder control. CST involvements produce weakness and reflex changes. A spinothalamic tract deficit produces the bilateral loss of pain and temperature sensation. Tactile, position, and vibratory sensation are normal. Urinary incontinence is usually present.

Dorsal cord syndrome results from the bilateral involvement of the dorsal columns, the corticospinal tracts, and descending central autonomic tracts to bladder control centers in the sacral cor. Dorsal column symptoms include gait ataxia and paresthesias. CST dysfunction produces weakness that, if acute, is accompanied by muscle flaccidity and hyporeflexia and, if chronic, by muscle hypertonia and hyperreflexia. Extensor plantar responses and urinary incontinence may be present.

A single anterior and two posterior spinal arteries supply the spinal cord. The anterior spinal artery supplies the anterior two-thirds of the cord. The posterior spinal arteries primarily supply the dorsal columns. The anterior and posterior spinal arteries arise from the vertebral arteries in the neck and descend from the base of the skull.

Various radicular arteries branch off the thoracic and abdominal aorta to provide additional blood supply to the spinal arteries.The largest and most consistently present of these radicular branches is the great ventral radicular artery or the artery of Adamkiewicz, which supplies the anterior spinal artery.

Up To Date

77
Q

A seven-year-old child is ventilated in the intensive care unit after an isolated closed head injury. Their serum sodium concentration is 142 mmol/L. The most appropriate intravenous maintenance fluid is:

a) 0.45% saline + 5% dextrose
b) 0.9% saline
c) CSL + 5% dextrose
d) CSL
e) 0.3% saline + 3% dextrose

A

STEPH b) 0.9% Normal Saline

Hypernatraemia is classified as:
Mild (146-149 mmol/L)
Moderate (150-169 mmol/L)
Severe (≥170 mmol/L)
–> stem not hypernatraemia

Head Injury
- Isotonic fluids (eg NaCl 0.9% recommended)

IV Fluids
Glucose 5% should be given in maintenance fluids for children with no other source of glucose –> likely enteral fed in this setting

**Also note: do not give hypotonic fluid
- Do NOT give glucose 4% with sodium chloride 0.18%
- Sodium chloride 0.45% solutions are only rarely indicated

RCH Guidelines
- Hypernatraemia
- Head Injury
- IV fluids

78
Q

You are anaesthetising an 18-year-old who has a Fontan circulation for exploratory laparotomy. They are intubated and ventilated with a ventilator that has been brought from the Intensive Care Unit. Their current arterial oxygen saturation is 70%. To improve oxygenation, you should INCREASE the:

a) Increase PIP
b) Increase PEEP
c) Increase inspiratory time
d) Increase expiratory time

A

D) increase expiratory time

Reworded repeat, but prev options don’t directly align with these

Low respiratory rates
- Short inspiratory time, long expiratory time (decreases time in which there is a high intrathoracic pressure)
- Low PEEP
- Tidal volumes: 5-6mL/kg
–> allows adequate pulmonary blood flow, normocarbia, low PVR
Avoid CHAOS (hypercarbia, hypothermia, acidosis, hypoxia, SVR drop)
Ideally spontaneous ventilation

Patients who have undergone the Fontan procedure depend on blood flow through the pulmonary circulation without the assistance of the right ventricle. The difference between central venous pressure and systemic ventricular end-diastolic pressure (termed the “transpulmonary gradient”) is the primary force promoting pulmonary blood flow and, more importantly, cardiac output.

Circulation in the Fontan patient is promoted by low pulmonary vascular resistance. Positive-pressure ventilation with increased tidal volumes, as described above, can result in excessive intrathoracic pressures, leading to decreased venous return to the heart and increased pulmonary vascular resistance.
In periods of low oxygen saturation, 100% inspiratory oxygen is appropriate.
The addition of PEEP will increase intrathoracic pressure, reducing venous return.
Trendelenberg positioning would increase CVP and therefore bloodflow through pulmonary circulation.

BJA: fontan circulation:
For relatively short procedures, Fontan patients are probably better off breathing spontaneously, as long as severe hypercarbia is avoided. For major surgery, or when prolonged anaesthesia is required, control of ventilation and active prevention of atelectasis is usually advisable. Potential disadvantages of mechanical ventilation in Fontan patients relate to the inevitable increase in mean intrathoracic pressure. This causes decreased venous return, decreased pulmonary blood flow, and hence, decreased cardiac output. Low respiratory rates, short inspiratory times, low PEEP, and tidal volumes of 5–6 ml kg−1 usually allow adequate pulmonary blood flow, normocarbia, and a low PVR. Hyperventilation tends to impair pulmonary blood flow, despite the induced respiratory alkalosis, because of the increased mean intrathoracic pressure.

https://academic.oup.com/bjaed/article/8/1/26/277637

79
Q

A 6-year-old child with a history of asthma is intubated and ventilated for tonsillectomy. During surgery, the SpO2 falls. You increase the FiO2 to 1.0 and hand-ventilate, and note that ventilation is difficult. The next step in the management is to:

a) Deepen anaesthesia
b) Give salbutamol
c) Ask surgeon to release gag
d) Suction ETT
e) Increase relaxant

A

STEPH c) Ask surgeon to release gag

Summary of Management of Lost Ventilation During Pediatric Tonsillectomy
* Manual ventilation with 100% oxygen, assessing compliance and leaks
* Ask surgeon to release mouth gag
* Bolus of propofol 2–3 mg/kg
* Consider LMA malposition, bronchospasm, circuit or filter problem, laryngospasm
* If no other cause apparent, assume laryngospasm and give suxamethonium 1–2 mg/kg as soon as SaO2 falls
* Lung recruitment breaths as SaO2 improves
* Deepen anesthesia, continue surgery

SIMS A Guide to Paediatric Anaesthesia

80
Q

You are called to assist with a patient in the intensive care unit who has had cardiac surgery three days ago and is now in cardiac arrest. External cardiac massage should aim for a systolic blood pressure of

a. 40
b. 60
c. 80
d. 100
e. 120

A

REPEAT

b. 60

BJA Article - ​Management of cardiac arrest following cardiac surgery - BJA Education

In the CICU, the effectiveness of ECC is confirmed by monitoring the arterial pressure trace with a target compression rate and depth to achieve a systolic impulse of > 60 mm Hg to maintain a mean perfusion pressure, preventing ventricular distension, LV wall stress, and ischaemia.

81
Q

A 65-year-old man is undergoing coronary artery bypass grafting. Immediately upon commencing cardiopulmonary bypass and prior to administering cardioplegia, the aortic line blood appears the same colour as the blood in the venous cannulae, and the low venous saturation alarm is activated on the bypass machine. The most appropriate management at this point is to:

a) Attach another oxygen tubing to oxygenator
b) Increase the oxygen mix with air:oxygen blender
c) Wean from bypass and ventilate lungs
d) Ventillate with 100% and continue

A

c) Ventilate and wean bypass

*Failure of oxygenation

I think wean bypass and ventilate - thoughts?

Asked about this at the Melbourne course - indicates failure of the oxygenator so agree ventilate and wean bypass, replace oxygenator. Steph.

82
Q

During rewarming on cardiopulmonary bypass, the most reliable surrogate for cerebral temperature measurement is:

A) Nasopharynx
B) Oxygenator arterial outlet
C) Oxygenator venous inflow
D) Bladder temp
E) PA Cath

A

A) Nasopharyngeal

Proximity to brain
Consistent correlation with core and brain temperature changes
Accessibility

Oxygenator blood temp represents temp of blood leaving circuit and doesnt reflect blood perfusing the brain

Clinical Techniques in Cardiovascular and Thoracic Surgery: This textbook discusses the monitoring of cerebral temperature during CPB and often cites nasopharyngeal temperature as a standard method due to its proximity to the brain.

Reference: Sabik, Joseph F., et al. “Temperature management and monitoring during cardiopulmonary bypass.” In: Clinical Techniques in Cardiovascular and Thoracic Surgery, edited by Little Brown and Company, 1998.
Perfusion: This journal article discusses various techniques for monitoring cerebral temperature during CPB, emphasizing the use of nasopharyngeal temperature probes.

Reference: Zollinger, Andreas, et al. “Temperature Management and Monitoring During Cardiopulmonary Bypass.” Perfusion, vol. 18, no. 1, 2003, pp. 3-9. doi:10.1191/0267659103pf582oa.

83
Q

The image below is from the transoesophageal echocardiogram of an adult patient who is about to undergo cardiac surgery. The structure labelled with the arrow is the:

TOE image - four chamber, with arrow pointing to leaflet closest to septum

A) Anterior mitral leaflet
B) Posterior mitral leaflet
C) Tricuspid septal leaflet
D) Tricuspud anterior leaflet
E) Tricuspid posterior leaflet

A

A) Anterior mitral leaflet

RV is an anterior structure hence anterior leaflet

84
Q

A patient presents for a trans-urethral resection of the prostate (TURP). He had a single drug-eluting coronary stent for angina pectoris inserted six months ago and is taking clopidogrel and aspirin. The most appropriate preoperative management of his medications is to:

a) Cease aspirin, continue clopidogrel
b) Cease aspirin for 10 days, cease clopidogrel for 5 days
c) Cease clopidogrel for 5 days, continue aspirin
d) Cease clopidogrel for 10 days, continue aspirin
e) Continue both aspirin and clopidogrel

A

REPEAT

c) Cease clopidogrel for 5 days, continue aspirin
- prostatic surgery, the risk of major bleeding may be greater than the risk of stent thrombosis
- For clopidogrel, we stop five days before surgery
- Clopidogrel, if stopped, should be restarted with a loading dose of 300 mg as soon as possible after surgery, perhaps later in the day if postoperative bleeding has stopped. Some experts recommend a higher loading dose of 600 mg to decrease time to effectiveness in the higher-risk postoperative setting
- suggest that surgery be performed in centers with 24-hour interventional cardiology coverage

UP TO DATE: Noncardiac surgery after PCI

Nonemergency noncardiac surgery — For patients who have undergone previous stenting with either BMS or DES and who will need cessation of one or both antiplatelet agents, we prefer to defer planned nonemergency, nonurgent noncardiac surgery until at least six months after stent implantation. The risks of noncardiac surgery before six months are increased after both BMS and DES.
For patients whose surgery requires cessation of one or both antiplatelet agents and cannot wait six months, and where the risks of delaying surgery outweigh the benefits, our recommended minimal duration of DAPT is four to six weeks, depending on the urgency of surgery and risk of thrombotic complication. This is based in part on evidence suggesting that the increased risk of MI and cardiac death is highest within the first month after stent placement and no clear difference in risk between BMS and DES. Although we prefer to wait at least six weeks when possible, in patients for whom earlier surgery is in their best interest after weighing risks and benefits, we sometimes refer patients as early as four weeks after stent placement.

The proinflammatory and prothrombotic risks of surgery may increase the baseline risk of stent thrombosis even in the presence of DAPT and regardless of stent type during this early period after stenting. We believe this risk to be higher prior to the minimum duration of DAPT recommended above, but the final decision to continue or discontinue antiplatelet therapy in the perioperative period should be made only after an informed discussion among the surgeon, managing cardiologist (and other health care providers), and patient has taken place. In many cases, DAPT can be continued in the perioperative period, although for some surgeries, such as neurosurgery, posterior eye surgery, or prostatic surgery, the risk of major bleeding may be greater than the risk of stent thrombosis.

In these patients who undergo noncardiac surgery before the recommended minimum duration of DAPT, a platelet P2Y12 receptor blocker should be discontinued for as brief a period as possible. Aspirin should be continued through the perioperative period, since the risk of stent thrombosis is further increased with the cessation of both aspirin and clopidogrel and surgery can usually be safely performed on aspirin. The rationale to continue aspirin comes in part from the POISE-2 trial (PCI subgroup analysis), which is discussed separately. However, as many neurosurgical patients, for whom bleeding might be life threatening or lead to severe adverse outcomes, were not enrolled in POISE-2, the optimal strategy is not known.

●Minor surgical and dental procedures usually do not require cessation of antiplatelet therapy.
●With regard to stopping P2Y12 inhibitor prior to noncardiac surgery, we generally follow recommendations found in the manufacturer’s package insert for each drug.
- For clopidogrel, we stop five days before surgery; that is, the last dose is taken on the sixth day before surgery.
- For prasugrel, we stop seven days before surgery.
- For ticagrelor, we stop three to five days before surgery.
- Some experts are willing to recommend shorter discontinuation periods for procedures less likely to be associated with major bleeding.
●Clopidogrel, if stopped, should be restarted with a loading dose of 300 mg as soon as possible after surgery, perhaps later in the day if postoperative bleeding has stopped. Some experts recommend a higher loading dose of 600 mg to decrease time to effectiveness in the higher-risk postoperative setting.
●We suggest that surgery be performed in centers with 24-hour interventional cardiology coverage

85
Q

A 45-year-old received a heart transplant one month ago. They develop a new supraventricular tachyarrhythmia without hypotension during gastroscopy. The most appropriate therapy is:

a) Adenosine
b) Amiodarone
c) Esmolol
d) Verapamil
e) Digoxin

A

REPEAT

d) Esmolol

Management of Arrhythmias After Heart Transplant
https://www.ahajournals.org/doi/10.1161/CIRCEP.120.007954

In asymptomatic patients, additional cardiac monitoring such as 24-Holter or an event monitor can be useful to assess the SVT burden, and a trial of atrioventricular nodal blockers (β-blockers preferably) can be attempted with caution in view of potential risk of bradycardia. Calcium channel blockers such as diltiazem and verapamil are contraindicated in patients taking immunosuppression such as tacrolimus and cyclosporine as it can impair the metabolism CYP3A, which increases the levels of these drugs potentially causing renal toxicity.

The use of adenosine in the management of SVT has remained a subject of controversy for over a quarter century. In the past, adenosine was contraindicated in patients post-OHT due to its supersensitivity and presumed risk of prolonged atrioventricular block.

Thus, based on the aforementioned data, in patients with OHT, adenosine is feasible and safe at reduced doses (starting at 1.5 mg for patients ≥60 kg) as long as patients are closely monitored, with dose escalation as needed. Furthermore, the 2010 American Heart Association guidelines on advanced cardiovascular life support also recommended lowering the initial dose of adenosine to 3 mg for the acute management of SVT in patients with OHT.

86
Q

According to the 5th National Audit Project (NAP5), the incidence of awareness during general anaesthesia using a non-relaxant technique with a volatile agent is approximately:

a. 1:700
b. 1:8000
c. 1:10000
d. 1:19000
e. 1:136,000

A

REPEAT
e. 1:136,000

https://www.bjanaesthesia.org/article/S0007-0912%2817%2930746-8/fulltext

1/670 E-LSCS
1/8000 with muscle relaxation
1/8600 CTS
1/8200 Volatile + neuromuscular blocking
Overall 1:19000

87
Q

An open Ivor-Lewis oesophagectomy is performed via a:

a Laparotomy then left thoracotomy
b Laparotomy, left neck incision
c Laparotomy, Right thoracotomy
d Left thoracotomy, left neck incision

A

REPEAT

C

Transhiatal - laparotomy & cervical anastomosis

Ivor-Lewis - laparotomy & R thoracotomy (tumour upper ⅔)

Thoracoabdominal - L throacotomy crossing costal margin & diaphragm (tumour lower ⅔)

Minimally invasive - thorascopic oesophageal mobilisation, laparoscopic gastric mobilisation & cervical
anastomosis

88
Q

A 69-year-old patient is dyspnoeic and complains of right shoulder tip pain while in the post-anaesthesia care unit after a laparoscopic-assisted anterior resection. A focused thoracic ultrasound is performed and an image of the right lung is shown below. This represents:

a) Pneumothorax
b) Pulmonary Oedema
c) Normal Lung
d) Consolidated Lung

A

REPEAT

c) Normal Lung

Normal lung = A lines (pleura) + batwing appearance + sliding

89
Q

According to the RELIEF study, in major abdominal surgery a liberal fluid strategy
(10 mL/kg of crystalloid at induction followed by 8 mL/kg/hour during the case)
compared to a restrictive fluid strategy, results in:

A. Increased bowel anastomosis breakdown
B. Increased mortality
C. Decreased mortality
D. No difference in wound infection
E. Decreased acute kidney injury

A

REPEAT

E. Decreased acute kidney injury

Restrictive had more AKI
Otherwise no outcome significant statistically

https://www.thebottomline.org.uk/summaries/relief/

90
Q

Soon after a peribulbar block, the patient’s eye rapidly becomes proptosed and tense, and the visual acuity is markedly decreased. A lateral canthotomy is indicated to:

a) Allow globe to continue to swell
b) Drain blood from behind eyeball
c) Allow the eye to proptose
d) Reduce pressure on the optic nerve

A

REPEAT

c) Allow the eye to proptose

Orbital Compartment Syndrome

The orbital compartment is a fixed space with limited capacity for expansion. If something like blood fills part of that space the pressure increases and may result in ischaemia of the optic nerve or the retina. A lateral canthotomy is a way of releasing this pressure.

You have up to approximately 2 hours before irreversible visual loss occurs. It may occur in less than 2 hours however, so speed is of the essence.

use local anesthetic but warn the patient that they may feel pain

Perform the canthotomy:
place the scissors across the lateral canthus and incise the canthus full thickness

Perform cantholysis:
Grasp the lateral lower eyelid with toothed forcepsPull the lower eyelid anteriorlyPoint the scissors toward the patient’s nose, place the blades either side of the lateral canthal tendon, and cut.

By cutting the canthal tendon,the counter pressure of the eyelid on the is relieved and the eye is allowed to proptose and pressure is relieved.

LITFL Goal of procedure: to release pressure on the globe & to decrease intraocular pressure enough to reinstitute retinal artery blood flow.

The canthotomy allows trhe eye to move forward and open up the space, reducing pressure. The globe itself should not swell.

91
Q

An 85-year-old is scheduled for open reduction and internal fixation of a fractured neck of femur today. They have no significant past medical history. Preoperative review including physical examination, full blood count, electrolyte profile and electrocardiogram performed yesterday were normal. In the anaesthetic bay, the monitor shows the patient to be in atrial fibrillation with a ventricular rate of 110 to 145 beats per minute. The blood pressure is 130/80 mmHg. The best initial treatment for the atrial fibrillation is:

A) Amiodarone
B) Metoprolol
C) Digoxin
D) Induce then cardiovert
E) Calcium Channel Blocker

A

B) Metoprolol

Guidelines from the American College of Cardiology/American Heart Association (ACC/AHA) and the European Society of Cardiology (ESC) recommend beta-blockers as a first-line therapy for rate control in atrial fibrillation.
Reference: January CT, Wann LS, Calkins H, et al. 2019 AHA/ACC/HRS focused update of the 2014 AHA/ACC/HRS guideline for the management of patients with atrial fibrillation: a report of the American College of Cardiology/American Heart Association Task Force on Clinical Practice Guidelines and the Heart Rhythm Society. Circulation. 2019;140(2)

92
Q

A ten-year-old boy (weight 30 kg) has a displaced distal forearm fracture that requires manipulation and application of plaster. The volume of 0.5% lidocaine that should be used for intravenous regional anaesthesia (Bier block) is:

a) 9ml
b) 12ml
c) 15ml
d) 18ml
e) 36ml

A

REPEAT

0.5% lidocaine = 5mg/ml
Max dose - 3mg/kg
30kg = 90mg max dose for this pt.
90/5=18mls of 0.5% solution
Therefore D.

93
Q

A patient for elective general anaesthesia has been noted to be chewing gum in the pre-operative area. The most appropriate course of action is to:

a) Delay 1 hour
b) Delay 2 hours
c) Delay 6 hours
d) Proceed
e) Cancel

A

d) Proceed

ANZCA PG07 appendix 1 - Chewing gum and boiled sweets should be discarded prior to inducing anaesthesia to avoid them being Inhaled as a foreign body but do not constitute an indication for delaying any procedure unless they have been ingested.

Therefore D

94
Q

An eight-year-old child with sickle cell disease is scheduled for emergency fixation of a fractured radius. Their preoperative haemoglobin (Hb) is 80 g/L. The most appropriate management is:

a) Proceed with careful haemostasis and check post op Hb
b) Transfuse to Hb >100
c) Blood type and screen
d) Exchange transfusion for HbSS <30%

A

REPEAT

b) transfuse for Hb >100
Emergency fixation means there is no time for an exchange transfusion

perioperative goals:
- planning and optimisation
- ensuring adequate O2 delivery
- hydration
- analgesia
- performed at a centre with a multidisciplinary sickle cell team

Children presenting for high-risk surgery (for example neurosurgical, cardiothoracic, or complex orthopaedic surgery) or high-risk children (previous stroke, acute CS, or end-organ damage), who were not included in this study, commonly receive an exchange transfusion or top-up transfusion, aiming for a preoperative haemoglobin concentration of 10 g dl−1 and Hb SS <30%. There is less evidence available for the role of transfusion in children with other forms of SCD.

Exchange transfusion vs. top-up transfusion
Exchange Transfusion:
- slowly removing the person’s blood and replacing with fresh donor blood or plasma
- Performed in cycles lasting a few minutes with slow removal of 5-20ml of blood and an equal amount of fresh pre-warmed blood or plasma flows into the person’s body
- in sickle cell disease blood is removed and replaced with donor blood to achieve a specific concentration of HbSS blood with a usual target of <30%
- Exchange transfusion removes HbS and increases HBA

Top-up transfusion:
- standard transfusion process of giving donor blood
- advantages of simple top-up include:
1. Increase oxygen carrying capacity
2. Decrease proportion of sickle haemoglobin HbS relative to Haemoglobin A (HBA)
3. Prevent or reverse complications of vast-occlusion
4. Can be given acutely
- disadvantages include:
1. Hyperviscosity if the Hb is increased to significantly over the patients baseline (target Hb should be 100g/l)
2. HbS is not removed, only diluted

95
Q

In neonates, an imaginary line joining the most superior points of the iliac crests will cross the spinal interspace of:

a) L3-4
b) L4-5
c) L5-S1
d) S1-S2

A

c) L5-S1

https://www.nysora.com/pediatric-atlas-of-ultrasound-and-nerve-stimulation-guided-regional-anesthesia/chapter34-spinal-anesthesia-preview/

96
Q

A normal systolic arterial blood pressure in the awake term neonate is approximately:

a) 60
b) 70
c) 85
d) 90

A

REPEAT

70

Term 1 hr - 70
Term 12 hr - 66
Day 1 asleep - 70
Day 1 awake - 71
Week 2 - 78
Week 4 - 85

According to RCH term 3.5kg SBP normal range is 60-95.

https://www.safercare.vic.gov.au/best-practice-improvement/clinical-guidance/neonatal/blood-pressure-disorders#goto-noninvasive-bp-measuring

97
Q

A 10-year-old child (weight 30 kg) presents to the emergency department in status epilepticus. They have received one dose of 15 mg midazolam buccally prior to arrival to hospital. According to Advanced Paediatric Life Support Australia guidelines the next drug treatment should be intravenous:

a) Ketamine 15mg IV
b) Midazolam 4.5mg IV
c) Propofol RSI and burst suppression
d) Levetiracetam 1.2g IV

A

REPEAT

b) Midazolam 4.5mg IV

Status epilepticus is defined as:
- Continuous seizure activity for 5 minutes or more without return of consciousness,
- recurrent seizures (2 or more) without an intervening period of neurological recovery

So needs urgent treatment.

APLS (in order)
Midaz dose is 0.15/kg IV/IM or 0.3mg buccal/IN

After first dose, if still seizing, repeat midaz, IV/IO

Levetiracetam of phenytoin
Lev = 40-60mg/kg, phen = 20mg/kg)

RSI

1st line: Midazolam IV/IO/IM –> 0.15mg/kg
2nd line: Midazolam IV/IO/IM –> 0.15mg/kg
3rd line: Keppra 40mg/kg (max 3g)
4th line: Phenytoin 20mg/kg or phenobarbitone
5th line: Intubation and deep sedation with midazolam, propofol +/- phenobarbitone

98
Q

A 5-year-old child with severe life-threatening anaphylaxis and no intravenous access, the recommended initial dose of intramuscular adrenaline is:

a. 100mcg
b. 150mcg
c. 300mcg
d. 500mcg
e. 600mcg

A

Repeat

b. 150mcg

Up to 6 years 150 IM
Over 6 years 300 IM

(ideally 10microg/kg)

Then commence adrenaline infusion 0.1mcg/kg/min to 2mcg/kg/min

Refractory management:
Additional IV fluid 20-40ml/kg,
Noradrenaline infusion 0.1- 2mcg/kg/min
Vasopressin infusion 0.02-0.06 units/kg/hr, glucagon 40mcg/kg IV

99
Q

A four-year-old child weighing 15 kg develops severe laryngospasm during an inhalational induction. Intravenous access is unobtainable. The recommended dose of intramuscular suxamethonium is:

a) 15mg
b) 30mg
c) 60mg

A

4 x 15 = 60mg

100
Q

In a patient who sustained significant burn injury, the blood concentration of propofol is:

a) Increased due to reduced cardiac output
b) Increased due to dehydration and reduced circulating volume
c) Reduced due to increased volume of distribution and clearance
d) Increased due to reduced renal clearance
e) Reduced due to increased inflammatory cytokines

A

REPEAT

c) Reduced due to increased volume of distribution and clearance

2010 Paper on major burns

The pharmacokinetic characteristics of a propofol bolus administered in patients with major burns were enhanced clearance and expanded volume of distribution.

BURN and WT were the important covariates. For sedation or anesthesia induction, a higher than recommended dose of propofol may be required to maintain therapeutic plasma drug concentrations in patients with severe burns.

Vigilance regarding the burned individual and careful titration of hypnotics to the desired effect cannot be overemphasized.

https://pubmed.ncbi.nlm.nih.gov/20510522/

101
Q

The following is a chest X-ray from a patient with dyspnoea after thoracic surgery.
The diagnosis is:

A. Dextracardia
B. Cardiac herniation
C. LLL collapse
D. Tension Pneumohorax

A

REPEAT

102
Q

A patient has blunt chest trauma. A thoracotomy is indicated if the immediate blood drainage after closed thoracostomy is greater than:

a) 500mL
b) 750mL
c) 1L
d) 1.2L
e) 1.5L

A

REPEAT

1,500 mL immediately

OR

200 mL/hr in the first 2-4 hours

103
Q

NP B lines (comet tails) in lung ultrasound are NOT observed in:

a) ARDS
b) Interstitial
c) Normal lung
d) Pneumothorax

A

D) pneumothorax

From BJA 2016 lung US article

The features of a pneumothorax are abolished sliding, absence of B lines, absence of the lung pulse, and presence of the lung point

Extra info from BJA

The US feature of Intersititial syndrome is B lines. These are artifacts generated by the juxtaposition of alveolar air and septal thickening (from fluid or fibrosis). Their characteristics are Occasional B lines can be seen in normal lungs (especially at the bases). Up to two between two adjacent ribs can be considered normal. Three or more between rib spaces (or close together in a transverse image) are pathological

104
Q

You have been asked to provide general anaesthesia for a complex thoracic endovascular aortic aneurysm repair. After the placement of a lumbar drain the recommended safe time before the administration of intravenous heparin is

a) 1 hour
b) 4 hours
c) 6 hours
d) 12 hours

A

AT - 1 hour

1.2.3 Time of lumbar drain placement to systemic intravenous heparinization should be greater than 60 minutes

Perioperative Management of Adult Patients with External Ventricular and Lumbar Drains Guidelines from the Society for Neuroscience in Anesthesiology and Critical Care

ASRA: 1 hour

Although the occurrence of a bloody or difficult neuraxial needle placement may increase risk, there are no data to support mandatory cancellation of a case. Direct communication with the surgeon and a specific risk-benefit decision about proceeding in each case are warranted.

Currently, insufficient data and experience are available to determine if the risk of neuraxial haematoma is increased when combining neuraxial techniques with the full anticoagulation of cardiac surgery. We suggest postoperative monitoring of neurologic function and selection of neuraxial solutions that minimise sensory and motor block to facilitate detection of new/progressive neurodeficits.

NYSORA:
Administration of intravenous heparin intraoperatively should be delayed for at least 1 hour after epidural placement; a delay before administration of subcutaneous heparin is not required. In cases of full heparinization for CPB, additional precautions include delaying surgery for 24 hours in the event of a traumatic tap, tightly controlling the heparin effect and reversal, and removing catheters when normal coagulation is restored.

105
Q

NP A 74-year-old presents for a femoral popliteal artery bypass procedure for peripheral limb ischaemia. Regarding its role in modifying their perioperative cardiovascular risk, clonidine:

a. Increased stroke
b. No change in complications
c. Increased death
d. Increased non fatal MI
e. Increased risk of non fatal cardiac arrest

A

REPEAT

e. Increased risk of non fatal cardiac arrest

POISE II
* clonidine 200mcg per day - did not reduce the rate of composite outcome of death or nonfatal MI - but it increased the risk of clinically important hypotension and nonfatal cardiac arrest
* aspirin initiation or continuation – no significant effect on rate of composite of death or non fatal MI but increased risk of major bleeding

Clonidine, as compared with placebo, was associated with an increased rate of nonfatal cardiac arrest

POISE 2 TRIAL

106
Q

NP A 65-year-old presents with an acute dissection of their thoracic aorta. Their blood pressure is 150/90 mmHg. The best medication to reduce the blood pressure is:

a) Esmolol
b) SNP
c) GTN
d) Hydralazine

A

A) esmolol

They get anti impulse therapy which usually starts off with beta blockade before alpha blockade.

Up to date: Patients often present with severe hypertension and are initially stabilized with fast-acting, intravenous beta blockers (eg, esmolol or labetalol) or calcium channel blockers. Anti-impulse therapy lowers blood pressure

107
Q

The strongest independent preoperative predictor of chronic postsurgical pain after knee arthroplasty is:

a) Anxiety
b) Depression
c) Catastrophising
d) Female

A

KATE
Catastrophizing

Page 24 APMSE
In children 8-18 yrs old “parent pain catastrophising” was main risk factor for development of CPSP

Significance of each risk factor varies with operation but pre existing psych factors (high state anxiety and pain magnification as a component of catastrophising) increases the risk across two types of surgery (TKR+Breast CA surgery)

108
Q

The analgesic drug with the most favourable Number Needed to Treat (NNT) for neuropathic pain is:

a) Amitriptyline
b) Gabapentin
c) Tramadol
d) Pregabalin
e) Carbamazepine

A

REPEAT reconsidered as Amitriptyline

a) Amitriptyline

https://www.bjaed.org/article/S2058-5349(18)30073-8/fulltext#tbl1

APMSE 5th edition:

Tramadol is an effective treatment for neuropathic pain with NNT of 4.4 (95%CI 2.9 to 8.8)

Alpha-2-delta ligands (gabapentinoids) are the only anticonvulsants with proven efficacy in the treatment of chronic neuropathic pain.
At doses of 1,800 mg to 3,600 mg/d, gabapentin is effective in treating neuropathic pain, in particular caused by postherpetic neuralgia (NNT 6.7; 95%CI 5.4 to 8.7)

Pregabalin
Postherpetic neuralgia: 300 mg/d pregabalin (NNT 5.3; 95%CI 3.9 to 8.1) (4 RCTs, n=713) and 600 mg/d (NNT 3.9; 95%CI 3.1 to 5.5) (4 RCTs, n=732);
* Painful diabetic neuropathy: 600 mg/d pregabalin (NNT 7.8; 95% CI 5.4 to 14) (5 RCTs, n=1,015);
* Mixed or unclassified post-traumatic neuropathic pain: 600 mg/d pregabalin (NNT 7.2; 95%CI 5.4 to 11) (4 RCTs, n=1,367);
* Central neuropathic pain (mainly SCI): 600 mg/d pregabalin (NNT 9.8; 95%CI 6.0 to 28) (3 RCTs, n=562).

Amitriptyline NNT 4.6 (TCAs are effective in treatment of neuropathic pain (amitrip NNT 4.6))
Amitriptyline

By order of favourable NNT:

  1. TCAs (amitriptyline) NNT: 3.6, NNH: 9
  2. Strong opioids NNT 4.3 NNH 11.7
  3. Tramadol NNT: 4.7, NNH 12.6
  4. SNRIs (duloxetine and venlafaxine) NNT 6.4, NNH 11.8
  5. Gabapentin NNT: 7.2 NNH 25.6
  6. Pregabalin NNT:7.7, NNH 13.9

ANZCA Pain book

Treatment of chronic neuropathic pain after SCI (Guy 2016 GL). These guidelines recommend:

  • First line: pregabalin, gabapentin and amitriptyline;
  • Second line: tramadol and lamotrigine (in incomplete SCI);
  • Third line: Transcranial direct current stimulation (tDCS) alone and combined with visual illusion;
  • Fourth line: TENS, oxycodone and dorsal root entry zone lesions.
109
Q

NP Self-report of pain in children is usually possible by the age of:

a. 2 yo
b. 4 yo
c. 6 yo
d. 8 yo

A

REPEAT

A) 4

4 yo = wong baker faces score 3-18.
8 yo = Visual analogue scale.

https://www.rch.org.au/rchcpg/hospital_clinical_guideline_index/Pain_assessment_and_measur ement/

APMSE 5 also

110
Q

A 30-year-old has had a free-flap operation of eight hours duration. They received an intraoperative remifentanil infusion and 10 mg morphine 30 minutes before the end of the operation. During recovery their pain score increased from 6/10 on arrival to 9/10 despite a further 10 mg of intravenous morphine. The most likely diagnosis is:

a. Acute behavioural change
b. OIH
c. Inadequate analgesia
D. Physical dependence

A

Nikki:
B)
Opioid induced hyperalgesia;
The key features are long case with Remi running, as well as increased pain following additional opioids.

https://associationofanaesthetists-publications.onlinelibrary.wiley.com/doi/full/10.1111/anae.13602

111
Q

NP Tranexamic acid is NOT useful in the management of:

A. Post cardiac bypass
B. Neurotrauma
C. PPH
D. Trauma
E. Upper GI bleed

A

REPEAT
E. Upper GI bleed

Incompressible sites, large volume blood loss and mortality risk are a few of the things that made GI bleeds seem like a natural fit for TXA administration. Early research seemed promising, but trials were small. The HALT-IT trial examined over 15,000 patients to see if TXA reduced death [14]. Not only did TXA have no effect on mortality, it increased the risk of seizure and thromboembolic events.

Take home: No demonstrated benefit with TXA in GI bleeding

112
Q

NP A drug that is contraindicated for a patient with a history of heparin induced thrombocytopaenia is:

a) Bivalirudin
b) Danaparoid
c) Prothrombinex
d) Fib conc

A

B) prothrombinex.

Prothrombinex product information states don’t give if hx of HITS

PROTHROMBINEX CONTAINS HEPARIN

113
Q

NP The use of intraoperative dexamethasone for tonsillectomy:

a) Increased oedema
b) Increased post tonsillectomy bleed
c) Increased Analgesic requirement
d) Reduced time to resumption of oral intake

A

REPEAT

d) Reduced time to resumption of oral intake

The effect of preoperative dexamethasone on early oral intake, vomiting and pain after tonsillectomy
https://pubmed.ncbi.nlm.nih.gov/15979735/

Conclusion: Preoperative dexamethasone use significantly reduces early posttonsillectomy pain, improves oral intake and facilitates meeting the discharge criteria while using standard anesthesia technique and sharp dissection tonsillectomy without any significant side effects.

114
Q

A patient experiences a postpartum haemorrhage associated with uterine atony that is unresponsive to oxytocin and ergometrine. The recommended intramuscular dose of carboprost (15-methyl prostaglandin F2 alpha) to be administered is:

a) 250mcg IM once
b) 250mcg IM q15mins, up to 2mg
c) 500mcg IM
d) 250mcg IV
e) 500mcg IV

A

REPEAT

b) 250mcg IM q15mins, up to 2mg

15-methyl-PGF2α (carboprost; Prostinfenem) which may be administered in one of two ways:

Intra-muscular injection of 0.25mg, in repeated doses as required at intervals of not less than 15 minutes to a maximum total cumulative dose of 2.0mg (ie up to 8 doses)

Source RANZCOG PPH Guideline 2021

115
Q

NP The oral morphine equivalent of tapentadol 50 mg (immediate release) is:

a) 5mg
b) 10mg
c) 15mg
d) 20mg
e) 25mg

A

B) 15mg

50mg x0.3

Tapentadol Conversion at 0.3
Tramadol conversion at 0.2
Oxycodone 1.5
Hydromorphone 5

Buprenorphine patch mcg/hr@2
Fentanyl patch mcg/hr @3

Oral Tapentadol 25mg = 8mg Oral Morphine

Oral Oxycodone 5mg = 8mg Oral Morphine

Oral Tramadol 25mg = Oral Morphine 5mg

Oral Hydromorphone 4mg = Oral Morphine 20mg

S/L Buprenorphine 200mcg = 8mg Oral Morphine

IV Oxycodone 5mg = Oral Morphine 15mg

IV Morphine 5mg = Oral Morphine 15mg

IV Hydromorphone 1mg = Oral Morphine 15mg

116
Q

A patient with known suxamethonium allergy is most likely to demonstrate cross reactivity with:

a. Mivacurium
b. Cisatracurium
c. Atracurium
d. Rocuronium
e. Cephazolin

A

REPEAT

AT - Rocuronium

Anaphylaxis to neuromuscular blocking drugs: incidence and cross-reactivity in Western Australia from 2002 to 2011 by Sadleir et al

(This paper was referenced in NAP 6 “Cross-sensitivity, based on skin testing and specific IgE, is common, with suxamethonium being the most commonly crossreacting drug (Sadleir 2013).”)

Fig 4 shows Rates of cross-reactivity for patients diagnosed with anaphylaxis according to the triggering NMBD.
- for sux anaphylaxis: highest cross-reactivity was roc (24%), then interestingly vec and cis were both tied at 12%, as were panc and atrac at 6%

PREVIOUS NOTES:

BJA Anaphylaxis to neuromuscular blocking drugs: incidence and cross-reactivity in Western Australia from 2002 to 2011
https://academic.oup.com/bja/article/110/6/981/245571

Rocuronium has a higher rate of IgE-mediated anaphylaxis compared with vecuronium, a result that is statistically significant and clinically important.

Cisatracurium had the lowest rate of cross-reactivity in patients who had previously suffered anaphylaxis to rocuronium or vecuronium.

Anaphylaxis rates (highest to lowest)
Primary anaphylaxis: rocuronium > atracurium > vecuronium > pancuronium = cisatracurium
Cross-reactivity: suxamethonium > rocuronium > vecuronium > pancuronium > atracurium > cisatracurium

117
Q

The correct blood collection tube for a mast cell tryptase test is a:

a. Potassium EDTA
b. serum separating tube
c. sodium citrate
d. sodium oxalate something

A

REPEAT

b. serum separating tube (gold top tube or red)

Potassium EDTA (purple)
-> FBC

sodium citrate (blue)
-> clotting screen/Rotem

sodium oxalate (green)
-> heavy metals (lead copper zinc)

118
Q

NP Once a unit of fresh packed red blood cells has been removed from controlled refrigeration the transfusion should be completed within

a) 2.5 hours
b) 3 hours
c) 3.5 hours
d) 4 hours

A

REPEAT

4 hours

As per Lifeblood

Start the transfusion as soon as possible after removing the blood component from approved temperature-controlled storage. Transfusion of each pack should be completed prior to the labelled expiry or within four hours, whichever is sooner.

Redcross: “Transfusion of each pack should be completed prior to the labelled expiry or within four hours, whichever is sooner. “

Shelf life of platelets: 5 days (Stored at 20-24 degrees, must be agitated gently and continuously)

FFP: Once FFP is thawed, must use within 24 hours.

Albumin administration: At RCH we allow the product to be administered within 6 hours of piercing the bottle. (from RCH.org)

Cryoprecipitate
Thawed cryoprecipitate should be maintained at 20°C to 24°C until transfused.
Once thawed, should be used within six hours if it is a closed single unit, or within four hours if it is an open system or units have been pooled.

119
Q

A previously healthy 22-year-old man is involved in an altercation and sustains a ruptured spleen. During splenectomy he is transfused with packed red blood cells. One hour into the transfusion his SpO2 rapidly decreases, his ventilator pressures
increase, frothy sputum appears in the endotracheal tube and he is febrile. The likely cause is:

a) TRALI
b) TACO
c) Rhesus incompatability
d) Anaphylaxis

A

AT

a) TRALI

Both TACO and TRALI are characterised by:
- hypoxia
- acute dyspnoea
- diffuse bilateral infiltrates

However, presence of fever is more in keeping with TRALI.

Reference:
Distinguishing between transfusion related acute lung injury and transfusion associated circulatory overload Robert C. Skeatea and Ted Eastlund

120
Q

The main difference between a size 5 microlaryngeal tube (MLT) and a standard size 5 endotracheal tube is that the size 5 MLT:

A. Smaller cuff
B. Longer length
C. Larger external diameter

A

Longer length

Different cuff size/ length: The MLT® has a cuff size/ length that would be typical for an adult-sized ‘standard’ ETT. A ‘standard’ pediatric 5.0 enndotracheal tube has a smaller cuff made for a pediatric-sized trachea (see picture below).

Distance of cuff from tube tip: In an MLT® the cuff is further away from the tube tip which is acceptable as the adult trachea is obviously longer than the pediatric one (see picture below).

121
Q

When using an endotracheal tube in an adult, the highest recommended cuff pressure to avoid mucosal ischaemia is

a. 10cmH2O
b. 20
c. 30
d. 40
e. 50

A

AT
REPEAT

C. 30cmH2O

(c.f. 60cmH20 for same question but for LMA cuff pressure)

References (a bit old now):
“Guidelines1,2 recommend a cuff pressure of 20 to 30 cm H2O. Inflation of the cuff in excess of 30 cm H2O damages the tracheal mucosa by compromising capillary perfusion. When pressures are greater than 50 cm H2O, total obstruction of tracheal blood flow occurs.3 In rare instances, massive overinflation of the cuff may lead to acute complications such as tracheal bleeding or rupture.4 “
- from Cuff Pressure of Endotracheal Tubes After Changes in Body Position in Critically Ill Patients Treated With Mechanical Ventilation by Lizy et al 2014

Whereby:
1 American Thoracic Society; Infectious Diseases Society of America. Guidelines for the management of adults with hospital-acquired, ventilator-associated, and healthcare-associated pneumonia. Am J Respir Crit Care Med. 2005; 171(4):388–416.

2 Lorente L, Blot S, Rello J. Evidence on measures for the prevention of ventilator-associated pneumonia. Eur Respir J. 2007;30(6):1193–1207.

https://link.springer.com/article/10.1007/s10877-020-00501-2

122
Q

The sensor on a NIM (Nerve Integrity Monitor) endotracheal tube used for thyroid surgery directly records

a. Electromyography of internal laryngeal muscles
b. Recurrent laryngeal nerve action potential
c. Movement of the vocal cords on the endotracheal tube
d. Pressure of the vocal cords on the endotracheal tube
e. Recurrent laryngeal nerve action potential

A

AT - REPEAT

a. Electromyography of internal laryngeal muscles

True - the NIM-EMG tube tests RLN function via EMG of the muscles

wiki:
Notably, the only muscle capable of separating the vocal cords for normal breathing is the posterior cricoarytenoid. If this muscle is incapacitated on both sides, the inability to pull the vocal folds apart (abduct) will cause difficulty breathing. Bilateral injury to the recurrent laryngeal nerve would cause this condition. It is also worth noting that all muscles are innervated by the recurrent laryngeal branch of the vagus except the cricothyroid muscle, which is innervated by the external laryngeal branch of the superior laryngeal nerve (a branch of the vagus).

Anaesthesia: Nerve Integrity Monitor tubes for thyroid surgery (2014)
Recurrent laryngeal nerve function should be monitored during thyroid surgery [1], either by direct observation of vocal cord function during surgery [2], which can be difficult, or by electromyography (EMG), during which electrodes are placed adjacent to the vocal cords to detect motion when the laryngeal nerves are stimulated. Electrodes can be secured to the outside of a tracheal tube so that they come into contact with the vocal cords during intubation [3]. Indeed, electrodes have been embedded into the material of the tracheal tube (Medtronic Nerve Integrity Monitor (NIM) Standard Reinforced EMG Endotracheal Tube, Medtronic Xomed Inc., Jacksonville, FL, USA), albeit leading to an increase in bulk and external diameter in comparison with equivalent internal diameter tubes (Fig. 2). The size and rigidity of the NIM tubes mandate oral tracheal intubation rather than nasal intubation, which itself is further inhibited by the positioning of cable attachments for the electrodes along the tube’s length.

http://smpp.northwestern.edu/bmec66/weightlifting/emgback.html
An electromyography (EMG) is a measurement of the electrical activity in muscles as a by product of contraction. An EMG is the summation of action potentials from the muscle fibers under the electrodes placed on the skin. The more muscles that fire, the greater the amount of action potentials recorded and the greater the EMG reading.

http://www.shanahq.com/main/content/reliable-technique-make-nim-tube-work-preview
The specialized endotracheal tubes (ETT), such as Xomed and TriVantage Nerve Integrity Monitoring (NIM) ETTs (Medtronic Xomed Inc., Jacksonville, FL USA) allow for RLN identification through continuous intraoperative EMG monitoring of the laryngeal muscles.

https://www.aana.com/newsandjournal/Documents/jcourse1_0410_p151-160_rev2.pdf
The Medtronic NIM electromyographic (EMG) endotracheal tube (Medtronic Xomed) is constructed of a flexible silicone elastomer and has a distal inflatable cuff.
The tube is fitted with 4 stainless steel wire electrodes (2 pairs) that are embedded in the silicone of the main shaft distance, slightly superior to the cuff. The electrodes are designed to make contact with the patient’s vocal cords to facilitate EMG monitoring of the RLN when connected to a multichannel EMG monitoring device. If monitoring correctly, the EMG monitor should show a consistent sound signal and an action potential tracing.

J Anaesthesiol Clin Pharmacol: The neural integrity monitor electromyogram tracheal tube: Anesthetic considerations (2013)
When attempting to identify LNs, a stimulating electrical current of 0.5-2.0 mA is used by the surgeon. This current is administered via a sterile probe, which is placed directly on the anatomical site in question. Additionally, return electrodes are positioned in the skin above the sternum.

When a LN is located, an electrical signal is subsequently generated by the motion of the vocal cords. An audibly recognizable “machine gun click” is then produced from the device’s associated monitor. This sound has a set frequency of 4 times/s (4 Hz). Simultaneously, an oscilloscope-like screen displays an identifiable sinusoidal response.

123
Q

Double sequential external defibrillation is performed by applying two shocks from:

a. Single set of pads, <1 second apart
b. Single set of pads, <5 seconds apart
c. Two sets of pads, <1 second apart
d. Two sets of pads, <5 seconds apart
e. Two sets of pads, simultaneously

A

AT

  • Two sets of pads, <1 second apart

(OR
- Two sets of pads, <5 seconds apart)

Following 3 standard shocks for refractory VF
Two defibrillators are used to provide sequential defibrillation with pads oriented in anterio-lateral and anterior posterior
The shocks are delivered near-simultaneously
- Anteriolateral first
- Then Anterioposterior

DOuble SEquential External Defibrillation for Refractory Ventricular Fibrillation (DOSE VF): study protocol for a randomized controlled trial
DSED: For paramedic services randomized to DSED, paramedics will apply a second set of defibrillation pads in the anterior-posterior configuration (Fig. 1) Application of the second set of defibrillation pads for the second defibrillator will occur during the 2-min cycle of CPR following the third defibrillation attempt, minimizing any interruptions in CPR. All subsequent defibrillation attempts will be carried out by sequential defibrillation shocks provided by two defibrillators. To ensure that shocks are not administered at the exact same moment, we will employ a short (less than 1 s) delay to provision of the second defibrillator shock. This will be accomplished by having a single paramedic pressing the “shock” button on each defibrillator in rapid succession as opposed to simultaneously. This technique will be performed across all sites when randomized to the DSED arm to maintain consistency in application within the trial.

NOTES ON PREVIOUS QUESTION 23.1

For DSED, to avoid possible defibrillator damage caused by shocks applied at the same instant, a short delay (<1 second) between shocks was created by having a single paramedic depress the “shock button” on each defibrillator in rapid sequence (anterior–lateral followed by anterior–posterior)

Among patients with refractory ventricular fibrillation, survival to hospital discharge occurred more frequently among those who received DSED or VC defibrillation than among those who received standard defibrillation.

https://www.nejm.org/doi/full/10.1056/NEJMoa2207304

124
Q

The initial management for a seizure during an awake craniotomy is:

a. Cold saline irrigation
b. Midazolam
c. Propofol

A

Nikki

A) cold saline irrigation

Intraoperative seizures have a higher incidence of transient motor deterioration and longer hospital stays.[10] First-line treatment should be irrigation of the brain with sterile iced saline. Propofol bolus (10 to 20 mg IV) or midazolam (1 to 2 mg IV) should be administered to terminate the seizure if iced saline is ineffective.

https://www.ncbi.nlm.nih.gov/books/NBK572053/

125
Q

A new antiemetic reduces the risk of post-operative vomiting by 20%. In a population with a baseline risk of post-operative vomiting of 10%, the number needed to treat is:

a. 2
b. 5
c. 10
d. 20
e. 50

A

AT
REPEAT

(base rate is 10%, experimental group is 8% (20% below 10%) therefore 100/ 2 = 50
or 1 divided by risk reduction

population risk = 10/100 patients get PONV
population risk + new antiemetic = 8/100 patients get PONV (8/100 as reduction by 20% with new drug)

RR= 0.10-0.08=0.02
NNT= 1/RR
=1/0.02
=50

126
Q

A risk factor which increases the likelihood of developing local anaesthetic systemic toxicity is:

a) Hypoxia
b) Alkalaemia
c) High alpha1-acid glycoprotein
d) Hypocarbia
e) Increased carnitine levels

A

AT
REPEAT

b) Hypoxia

Hypoxia

Local anaesthetics are bases with pKa above physiological pH. The more alkalaemic the environment the more unionionised (B) form there is – which will speed diffusion across plasma membrane = can exert Na+ channel blockade.

https://www.bjanaesthesia.org/article/S0007-0912(17)38238-7/pdf
https://academic.oup.com/bjaed/article/15/3/136/279390
https://www.ncbi.nlm.nih.gov/pmc/articles/PMC6087022/

Hypoxia – metabolic acidosis = ion trapping = increased toxicity
Alkalaemia = prevents ion trapping in tissues (intralipid can work) = reduced toxicity
High a1GP = reduced free fraction (a1gp high affinity, low capacity) = reduced toxicity
Low CO2 = alkalosis = prevents ion trapping in tissues (intralipid can work) = reduced toxicity
Carnitine deficiency = increased toxicity, therefore increased carnitine will reduce toxicity https://pubmed.ncbi.nlm.nih.gov/19849674/

a. Hypoxia - Yes
b. Alkalemia - No - acidosis causes increased ionised fraction due to its weak base properties
c. High α1-acid glycoprotein - No, normally bound to alpha-1 acid glycoprotein
d. Hypocarbia < (decreased seizure threshold) - No - hypercarbia increases CNS blood flow and increases risk of seizures due to more LA delivered to CNS
e. Increased carnitine levels -s - Never heard of it

127
Q

The image below shows results from non-inferiority trials. The trial labelled ‘M’ is best described as:

a) Non inferiority graph (line crossed the 0 line but not non-inferior dotted line)

A

NIKKI

128
Q

A 30-year-old athlete undergoing a knee arthroscopy under general anaesthesia
develops intraoperative tachycardia. A 12-lead electrocardiogram is obtained and
shown below. The most likely diagnosis is:

a) AF
b) Flutter
c) AVNRT
d) Multifocal atrial tachycardia

A

AT

Repeat

Delta waves present, therefore WPW = AVRT

WPW + delta wave = AVRT → anatomical re-entry circuit (Bundle of Kent)
AVNRT is a functional re-entry circuit within the AV node
ECG features of AVNRT
● Regular tachycardia ~140-280 bpm
● Narrow QRS complexes (< 120ms) unless there is co-existing bundle branch block, accessory pathway, or rate-related aberrant conduction
● P waves if visible exhibit retrograde conduction with P-wave inversion in leads II, III, aVF. They may be buried within, visible after, or very rarely visible before the QRS complex
https://litfl.com/supraventricular-tachycardia-svt-ecg-library/

129
Q

Analysis of variance (ANOVA) is a statistical test to determine:

a) The validity between an expected and observed outcome in a population
b) The difference between the means of more than two populations
c) The difference between two populations with non-parametric data
d) The degree of similarity of the median between two or more populations
e) If the variance within a population is likely to be abnormally or normally distributed

A

REPEAT

B) analyse the difference between the means of more than two groups

130
Q

When performing a brachial plexus block at the level of the axilla, the structure indicated by the arrow is the:

a) Musculocutaneous
b) Radial
c) Ulnar

A

REPEAT

131
Q

A third heart sound at the apex may be heard in:

a) Healthy people aged less than 40
b) Mitral prolapse
c) HTN

23.1 OPTIONS:

a) pulmonary stenosis
b) pulmonary hypertension
c) pericarditis
d) pregnancy

A

AT

Repeat

Can occur in healthy young people

The third heart sound is mainly created by the early-diastolic rapid distension of the left ventricle that accompanies rapid ventricular filling and abrupt deceleration of the atrioventricular blood flow

S3 may be normal in people under 40 years of age and some trained athletes but should disappear before middle age. Re-emergence of this sound late in life is abnormal[5] and may indicate serious problems such as heart failure.

‘Sounds like Ken-tu-cky’

132
Q

In pulmonary function testing the presence of airflow limitation is defined by a post- bronchodilator FEV1/FVC ratio less than:

a) 0.5
b) 0.6
c) 0.7
d) 0.8

A

c) 0.7

133
Q

Local anaesthetic-induced myotoxicity is most likely to be associated with:

A. Biers
B. Interscalene
C. Sciatic
D. Adductor Canal

A

REPEAT

D. Adductor Canal

unclear phenomonenon
prolonged exposure and high concentrations of local anaesthetic

134
Q

Regarding healthcare research, the PICO framework describes:

a) Critical appraisal
b) Meta-analysis
c) Observational study
d) Systematic review

A

REPEAT

a) Critical appraisal

PICO is a mnemonic used to describe the four elements of a good clinical foreground question:

P = Population/Patient/Problem - How would I describe the problem or a group of patients similar to mine?

I = Intervention - What main intervention, prognostic factor or exposure am I considering?

C = Comparison - Is there an alternative to compare with the intervention?

O = Outcome - What do I hope to accomplish, measure, improve or affect?

135
Q

The 12-lead electrocardiogram shown is most consistent with acute total occlusion
of the:

a) LAD
b) PDA
c) OM
d) RCA

A

a) LAD

Am I missing something? I can’t see total occlusion of anything here.

Wellens syndrome- Lad

https://litfl.com/wellens-syndrome-ecg-library/

136
Q

A 70-year-old patient booked for a revision total hip replacement is reviewed in preadmission clinic ten days before surgery. The following blood test results are

noted:
haemoglobin 110 g/L
ferritin 51 mcg/L
CRP (c-reactive protein) 10 mg/L

The most appropriate management for this patient should be to:

a Transfuse 2u pRBC
b Give oral iron therapy and continue with surgery
c Give oral iron therapy and defer surgery for 6 weeks
d Give IV iron
e Do nothing

A

c Give oral iron therapy and defer surgery for 6 weeks

IV Iron is probably best, however all in agreement that surgery should be delayed making C the correc t choice based on guidlines accepting the flaws in oral iron therapy

https://www.blood.gov.au/module-2-perioperative-patient-blood-management-guidelines

Iron therapy
Oral iron in divided daily doses. Evaluate response
after 1 month. Provide patient information
material.
IV iron if oral iron contraindicated, is not tolerated
or effective; and consider if rapid iron repletion
is clinically important (e.g. <2 months to non
deferrable surgery).
NOTE: 1 mcg/L of ferritin is equivalent to 8–10 mg
of storage iron. It will take approximately 165 mg of
storage iron to reconstitute 10 g/L of Hb in a 70 kg
adult. If preoperative ferritin is <100 mcg/L, blood
loss resulting in a postoperative Hb drop of ≥30 g/L
would deplete iron stores.
In patients not receiving preoperative iron therapy,
if unanticipated blood loss is encountered,
150 mg IV iron per 10g/L Hb drop may be given
to compensate for bleeding related iron loss
(1 ml blood contains ~0.5 mg elemental iron

Footnotes
1 Anaemia may be multifactorial, especially in the elderly or in those with chronic disease, renal impairment,
nutritional deficiencies or malabsorption.
2 In an anaemic adult, a ferritin level <15 mcg/L is diagnostic of iron deficiency, and levels between
15–30 mcg/L are highly suggestive. However, ferritin is elevated in inflammation, infection, liver disease and
malignancy. This can result in misleadingly elevated ferritin levels in iron-deficient patients with coexisting
systemic illness. In the elderly or in patients with inflammation, iron deficiency may still be present with
ferritin values up to 60–100 mcg/L.
3 Patients without a clear physiological explanation for iron deficiency (especially men and postmenopausal
women) should be evaluated by gastroscopy/colonoscopy to exclude a source of GI bleeding, particularly a
malignant lesion. Determine possible causes based on history and examination; initiate iron therapy; screen
for coeliac disease; discuss timing of scopes with a gastroenterologist.
4 CRP may be normal in the presence of chronic disease and inflammation.
5 Consider thalassaemia if MCH or MCV is low and not explained by iron deficiency, or if long standing.
Check B12/folate if macrocytic or if there are risk factors for deficiency (e.g. decreased intake or absorption),
or if anaemia is unexplained. Consider blood loss or haemolysis if reticulocyte count is increased. Seek
haematology advice or, in presence of chronic kidney disease, nephrology advice

137
Q

A 55-year-old with no past history of ischaemic heart disease is three days post-total hip replacement surgery. They have an episode of chest pain at rest with features typical of angina that lasts 30 minutes before fully resolving. There are no electrocardiogram changes and no troponin rise. The diagnosis is

a. No diagnosis made
b. Unstable angina
c. STEMI
d. NSTEMI
e. MINS

A

REPEAT
b. Unstable angina

UTD:

Unstable angina (UA) and acute non-ST elevation myocardial infarction (NSTEMI) differ primarily in whether the ischemia is severe enough to cause sufficient myocardial damage to release detectable quantities of a marker of myocardial injury (troponins):

●UA is considered to be present in patients with ischemic symptoms suggestive of an ACS and no elevation in troponins, with or without electrocardiogram changes indicative of ischemia (eg, ST segment depression or transient elevation or new T wave inversion).

●NSTEMI is considered to be present in patients having the same manifestations as those in UA, but in whom an elevation in troponins is present.

MINS: Myocardial injury after non-cardiac surgery (up to 30 days post-op):
1. Elevated postop troponin
2. Resulting from myocardial ischaemia (i.e. no evidence of a non-ischaemic aetiology), not requiring an ischaemic feature (i.e. no chest pain, no ECG change)

VISION studies (Vascular Events in Noncardiac Surgery Patients Cohort Evaluation) demonstrated that severity of MINS strongly associated with 30-day mortality after NCS.

hs-cTnT
<20ng/L ~ 0.5% 30 day mortality
20-64ng/L ~3% 30 day mortality
65-999 ng/L ~9% 30 day mortality
>1000ng/L ~30% 30 day mortality

Whilst VISION trial identified MINS in at risk patients, the question now becomes what interventions are available to prevent this complication?

138
Q

The QRS axis of the attached electrocardiograph is closest to:

a) -90
b) -45
c) +45
d) +90

A

VICTORIA

B

https://litfl.com/super-axis-man-sam/

139
Q

In septic shock, the recommended target mean arterial pressure in an adult is:

a) 50 mmHg
b) 55 mmHg
c) 60 mmHg
d) 65 mmHg

A

VICTORIA

Surviving Sepsis 2021 guidelines

D

140
Q

Kate
A 50-year-old patient with carcinoid syndrome undergoing resection of a peripheral hepatic metastasis develops a sudden fall in blood pressure from 110/70 mmHg to 85/50 mmHg without significant bleeding. The most appropriate management is:

a. Normal saline bolus
b. Octreotide 50mcg bolus
c. Metaraminol 0.5mg
d. Noradrenaline 5mcg bolus
e. Calcium 6.8mmol

A

REPEAT

b. Octreotide 50mcg bolus

Vasoactive hormone release intra-operatively is best treated with intravenous boluses of 20–50 µg of octreotide, titrated to haemodynamic response. Vasopressin as an alternative vasoconstrictor that may be useful if prolonged vasoconstriction is required; however, the evidence base is small.

It must be borne in mind that concomitant fluid losses, especially bleeding, may be responsible for intra-operative instability rather than hormone excess and that fluid resuscitation may be the answer rather than further octreotide therapy

https://academic.oup.com/bjaed/article/11/1/9/285683

141
Q

Kate
In cardiac surgery a low-normal central venous pressure and a low blood pressure with a hyperdynamic heart is suggestive of:

a) Vasoplegia
b) Hypovolaemia

A

A) vasoplegia
Vasoplegia is characterized by a normal or augmented cardiac output with low systemic vascular resistance (SVR) causing organ hypoperfusion. The exact definition has varied but typically is considered when shock occurs within 24 h of CPB in the setting of a cardiac index (CI) is greater than 2.2 L/kg/m2 and SVR less than 800 dyne s/cm5

142
Q

Kate
One metabolic equivalent (1MET) is defined as the:

a) O2 consumption during walking 4km/h
b) O2 consumption at rest
c) Energy consumption while walking at 4km/h
d) Energy consumption during rest

A

REPEAT

B) 02 consumptiom at rest

One metabolic equivalent (MET) is the amount of oxygen consumed while sitting at rest, equal to 3.5 mL O2 per kg body weight × min

143
Q

Kate According to the Australian and New Zealand Committee on Resuscitation guidelines, the minimum distance a defibrillation pad should be placed away from a pacemaker or implantable cardiac defibrillator generator is:

a) 8cm
b) 12cm
c) 16cm

A

A) 8cm

ANZCOR:
In patients with an ICD or a permanent pacemakerthe defibrillator pad/paddle is placed on the chest wall ideally at least 8 cm from the generator position

144
Q

Kate A 25-year-old sustains a burn to 30% of their total body surface area. A physiological change expected within the first 24 hours is:

a) Increased CI
b) Decreased SVR
c) Increased PVR
d) Increased hepatic blood Flow

A

REPEAT

C) Increased PVR

Initial - may lead to burns shock

Increased SVR (by 200%)
Increased PVR
Increased capillary permeability- loss of intravascular volume/Increased interstitial oedema
Reduced CO
SIRS

By day 5 hyperdynamic circulation
Tachycardia
Increased CO (remains for at least 2 years maybe longer)

145
Q

When inadvertent total spinal anaesthesia occurs in an awake neonate, the first sign is most likely to be

a) Decreased HR
b) Decreased BP
c) Desaturation
d) Loss of consciousness

A

MAYANK - found some options for you! Steph
? Options

c) Desaturation

Total spinal anesthesia has been reported in neonates. It is most commonly manifested by apnea with no change in systemic blood pressure or heart rate, although should pronounced bradycardia occur, a reduction in cardiac output is likely and should be treated aggressively. It can occur after a dose of as little as 0.6 mg/kg of tetracaine. Alteration in position, particularly by raising the lower body above the level of the head or thorax, may be the most common cause of a high spinal block. Although the rate of administration of the local anesthetic does not appear to affect the level of spinal anesthesia in adults, similar studies have not been conducted in neonates or infants. It is possible that factors, such as the use of a relatively large-bore needle (22-gauge) and a tuberculin syringe providing the means for injecting with high pressure, along with the small distance between vertebrae, combines to make the rate of injection an important consideration in neonates and infants by producing unintended barbotage. We have observed this complication with rapid drug administration. Management consists of assisted or controlled ventilation until the return of spontaneous respiratory function.

Cote. Practice of Anesthesia for Infants and Children, A, Seventh Edition - confirmed by QCH consultant

**I initially thought b) Decreased BP

Significant changes in blood pressure are uncommon in pediatric patients after the accurate administration of epidural analgesia. A high sympathetic single-injection caudal block to T6 has been found to evoke no significant changes in heart rate, cardiac index, or blood pressure in children. Even when thoracic epidural block is combined with general anesthesia, cardiovacular stability is usually maintained in otherwise healthy pediatric patients. Therefore, hypotension should raise the suspicion of total spinal anesthesia and/or an intravascular injection leading to local anesthetic toxicity. After these complications are ruled out, other causes, such as hydration status, intravascular filling pressure, and inotropic state, and the depth of anesthesia should be assessed.

NYSORA

146
Q

Kate A bleeding patient has ROTEM results including: [table attached]. The most
appropriate treatment is:

a) Fibrinolysis

A
147
Q

Kate

A 54-year-old has a laryngeal mask airway inserted for a surgical procedure. The
following day it is noted that the tongue is deviated to the right. The most likely site
of nerve injury is the right:

a) Hypoglossal

A

REPEAT

Hypoglossal (deviates to the affected side)

Nerve injuries : (pressure neuropraxia)
Lingual nerve injury (most common)
RLN (most serious)
Hypoglossal
Glossopharyngeal
Inferior alveolar
Infra orbital
Usually self resolve except for RLN

148
Q

Kate The nerve marked by the arrow is the:

A

REPEAT

Axillary Nerve

149
Q

Kate
In an adult weighing 70 kg, a bedside assessment of haemodynamic status shows a left ventricular end-diastolic diameter of 2.4 cm. This finding suggests:

a) Hypovolaemia
b) Normal
c) Hypervolaemia

A

Hypovolaemia
Normal for end diastole is 3.5 to 5.6cm

Image
https://thoracickey.com/cardiac-chambers/

PSAX End diastolic AREA:
Hypovolemia <8cm2
Normal 8-14cm 2
Hypervolemia > 14cm2
IVSd and IVSs – Interventricular septal end diastole and end systole. The normal range is 0.6-1.1 cm.
LVIDd and LVIDs – Left ventricular internal diameter end diastole and end systole. The normal range for LVIDd is 3.5-5.6 cm, and the normal range for LVIDs is 2.0-4.0 cm.
LVPWd and LVPWs – Left ventricular posterior wall end diastole and end systole. The normal range is 0.6-1.1 cm.
RVDd – Right ventricular end diastole. The normal range is 0.7-2.3 cm.
Ao Root Diam – Aortic root diameter. The normal range is 2.0-4.0 cm.
LA Diameter – Left atrium diameter. The normal range is 2.0-4.0 cm.
The IVSd and IVPWd measurements are used to determine left ventricular hypertrophy, which is the thickening of the muscle of the left ventricle. LV hypertrophy is a marker for heart disease. In general, a measurement of 1.1-1.3 cm indicates mild hypertrophy, 1.4-1.6 cm indicates moderate hypertrophy, and 1.7 cm or more indicates severe hypertrophy.

Hypovolaemia
Normal for end diastole is 3.5 to 5.6cm

150
Q

Kate For driving pressure guided ventilation, driving pressure is the:

a) Pplat-peep
b) Peak pressure-peep
c) Other formulas

A

Pplat-PEEP

driving pressure is defined as distending pressure above the applied Peep Required to generate Vt
- key variable for optimisation when performing mechanical ventilation in ARDS
- also Vt/CRS (Ratio of Tidal volume to static resp system compliance)